You are on page 1of 64

353

EDITORIAL
Va lav
 Linek

Many CRUX with Mayhem readers have ommented on the urrent


ba klog of four-digit problem proposals (for those problems that appear near
the end of ea h issue). The ba klog of these is due to the fa t that we pub-
lish only 100 four-digit problems per year, but we re eive many more than
100 problem proposals per year. This situation is daunting, espe ially for
new proposers, but also for established proposers who may have a favourite
problem languishing in a sta k.
To at least partially address this situation, any new proposer may now
submit up to two (2) problem proposals and request that they be expedited,
meaning that we will do our best to evaluate those proposals as soon as
possible. In addition, anyone who has already proposed problems to CRUX
may also submit two proposals to be expedited. To help us implement this
new poli y, we advise proposers to refer to the proposal numbers that we
give out whenever a problem is proposed. This makes it mu h easier for us
to tra k your problem proposal, whereas without a proposal number we may
have to sear h literally hundreds of pages of paper in dozens of le folders.
We will also begin streaming the problem proposals that we re eive into
ategories, namely: Algebra and Number Theory (a single ategory), Logi ,
Cal ulus, Combinatori s, Inequalities (in luding Geometri Inequalities),
Geometry, Probability, and (if any of these do not t!) Mis ellaneous Prob-
lems. Currently we have a large store of proposed inequalities, so we are
parti ularly interested in re eiving problem proposals in the other ategories
in order to a hieve a balan e of topi s.
In another department, our Contributor Pro les will be resuming in
2010 and we have re eived at least two re ommendations so far this year (any
time is a good time to re ommend someone to be pro led and we wel ome
more re ommendations).
Lastly, we ask that all solvers begin ea h new solution on a separate
page, with their name and onta t information at the top of the page. In
parti ular our Mayhem editor, Ian VanderBurgh, makes a spe ial appeal to
all Mayhem solvers to follow this pro edure.
354

SKOLIAD No. 119

Lily Yen and Mogens Hansen

Please send your solutions to problems in this Skoliad by 1 January, 2010. A


opy of Crux will be sent to one pre-university reader who sends in solutions
before the deadline. The de ision of the editors is nal.
Our ontest this month is the World Youth Mathemati s Inter ity
Competition, Individual Contest, Part I, 2005, whi h was held in Kaosiung,
Taiwan. We thank Wen-Hsien Sun, Chiu Chang Mathemati s Edu ation
Foundation, Taipei, Taiwan, for providing us with this ontest and for per-
mission to publish it. We also thank Rolland Gaudet, University College of
Saint-Bonifa e, Winnipeg, MB, for translation of this ontest.

Competition
 mathematique
 mondiale des jeunes
Con ours individuel, Partie I, 2005

Temps limite : 2 heures

1 . La somme d'un entier a 4 hi res et de ses 4 hi res est 2005. Quel est
et entier a 4 hi res ?
2. Dans le triangle ABC , on a AB = 10 et AC = 18 ; M est le point milieu
de BC ; la ligne passant par M et parallele
 a la bisse tri e de ∠CAB oupe
AC au point D . Determiner
 la longueur de AD.
3 . Soient x, y et z des nombres positifs tels que x+y+xy = 8, y+z+yz = 15
et z + x + zx = 35. Determiner
 la valeur de x + y + z + xy.
4. Au total, le nombre de hampignons ueillis par 11 gar ons et n lles est
egal
 a n2 + 9n − 2, ou haque personne a ueilli exa tement le m^eme nombre
de hampignons. Determiner
 l'entier positif n.
5. L'entier positif x est tel que x et x + 99 sont tous les deux des arres.

Determiner
 la somme de tous les entiers x ayant la m^eme propriet
 e.

6. Les ot
^ es
 d'un triangle re tangle sont tous les deux des entiers positifs ;
un de es ot^ es
 est de longueur au plus 20. Le rapport du rayon du er le
ir ons rit au rayon du er le ins rit de e triangle est de 5 : 2. Determiner

la valeur maximale du perim
 etre
 de e triangle.
7. Soit α la plus grande ra ine de (2004x)2 − 2003 · 2005x − 1 = 0 et soit
β la plus petite ra ine de x2 + 2003x − 2004 = 0. Determiner
 la valeur de
α − β.
355

1
8. Soit a un nombre positif tel que a2 + a2
= 5. Determiner
 la valeur de
1
a3 + 3 .
a

9 . Dans la gure, ABCD est un re tangle D ...................................................................................................................................... C


ave AB = 5 et tel que le demi er le de
... .. ...... .
... ........ ........ ............. ....
... ... .........
. ... ... ..
diametre
 AB oupe CD en deux points.
.
...
..
... .. ......... ... ... ..
...... ........ ... .....
...... ................
Si la distan e d'un e es points a A est 4,
... ......
... .......... ......
...........................................................................................
determiner
 la surfa e de ABCD. A B
  n 2 n−1 
. Soit a donne par 9 n 10 10 10 10
ou n
 
10 −1− − − ··· −
9 9 9 9
est un entier positif. Si a est un entier, determiner
 la valeur maximale de a.
11. Dans un entier a  deux positions de imales,
 le hi re en position dix est
plus grand que le hi re en position un ; le produit de es deux hi res est
divisible par leur somme. Quel est et entier a deux positions de imales
 ?
P ..............................B S
. Dans la gure, P QRS est un re -
............................................................
12 .... . ... ...
.. ................... ...
..
tangle de surfa e 10. A est un point
... ....
..........
.......... ...
.. .. .......... ....
...
sur RS et B est un point sur P S , tels A
. .............
... ..
.. ... ............ .....
que la surfa e du triangle QAB est 4.
.. ... ..
..........
.......... ...
.. .. ............ ...
... .. .............
Determiner
 la plus petite valeur possible
...
...............................
...............................................................................................
pour P B + AR. Q R

World Youth Mathemati s Inter ity Competition


Individual Contest, Part I, 2005

2 hours allowed

1 . The sum of a four-digit number and its four digits is 2005. What is this
four-digit number?
2 . In triangle ABC , AB = 10 and AC = 18. M is the midpoint of BC ,
and the line through M parallel to the bise tor of ∠CAB uts AC at D. Find
the length of AD.
3. Let x, y , z be positive numbers su h that x+y +xy = 8, y +z +yz = 15,
and z + x + zx = 35. Find the value of x + y + z + xy.
4 . The number of mushrooms gathered by 11 boys and n girls is n2 + 9n − 2,
with ea h person gathering exa tly the same number. Determine the positive
integer n.
5. The positive integer x is su h that both x and x + 99 are squares of
integers. Find the sum of all su h integers x.
356

6 . The side lengths of a right triangle are all positive integers, and the length
of one of the legs is at most 20. The ratio of the ir umradius to the inradius
of this triangle is 5 : 2. Determine the maximum value of the perimeter of
this triangle.
7. Let α be the larger root of (2004x)2 − 2003 · 2005x − 1 = 0 and β be the
smaller root of x2 + 2003x − 2004 = 0. Determine the value of α − β .
8 . Let a be a positive number su h that a2 + a12 = 5. Determine the value
of a3 + a13 .

9 . In the gure, ABCD is a re tangle D ...................................................................................................................................... C


with AB = 5 su h that the semi ir le with
... .. ... .... .
... ........ ......... ............ .....
... ... ......... ... ... ..
diameter AB uts CD at two points. If
...
...
. ... ... ..
... .. ........
...... ........ ... .....
..... ................
the distan e from one of them to A is 4,
... ......
... .......... ....
.............................................................................................
nd the area of ABCD. A B
  n  2  n−1 
10 10 10 10
10 . Let a be 9 n
9
−1− −
9
− ··· −
9
where n is 9
a positive integer. If a is an integer, determine the maximum value of a.
11 . In a two-digit number, the tens digit is greater than the ones digit. The
produ t of these two digits is divisible by their sum. What is this two-digit
number?
P ..............................B
............................................................
S
... . .. ...
.. ...................
. In the gure, P QRS is a re tangle
...
12 ..
... ..........
... .... ..
of area 10. A is a point on RS and B is
..........
.. .. .......... .....
..........
A
... . .
a point on P S su h that the area of tri-
. ..
... ... ........... ..
.. ... ............. .....
............
angle QAB is 4. Determine the smallest
.. .. ..
...
...
... ...
... .. .
........... ...
...... ........................
possible value of P B + AR.
.
..................................................................................................
Q R

Next follow the solutions to the National Bank of New Zealand Junior
Mathemati s Competition 2005 [2008 : 385{391℄.
1 (For year 9, Form 3 only). Note: In this question the word \digit" means a
positive single-digit whole number, that is, a member of the set {1, . . . , 9}.
On a long journey by ar, Mi hael was starting to get bored. To keep him
amused, his mother asked him some arithmeti questions. The rst question
she asked was \Can you think of ve di erent digits whi h add to a multiple
of 5?" Mi hael answered straight away \That's easy Mum. 1, 2, 3, 4, and
5 work be ause 1 + 2 + 3 + 4 + 5 = 15, and 15 is a multiple of 5 be ause
15 = 5 × 3."
357

Now answer the other questions whi h Mi hael's mother asked.


(a) Write down a set of ve di erent digits whi h add to 35.
(b) Write down a set of three di erent digits whi h add to a multiple of 5,
but whi h don't in lude 5 itself or 1.
( ) How many di erent sets of four di erent digits are there whi h add to
a multiple of 5, but whi h don't in lude 5 itself or 1? (Note that writing
the same set of numbers in a di erent order doesn't ount here.)
(d) Is it possible to write down a set of six di erent digits whi h add to a
multiple of 5, but whi h don't in lude 5 itself? If it is possible, write
down su h a set. If it is not possible, explain brie y why it annot be
done.
(e) Is it possible to write down a set of seven di erent digits whi h add to
a multiple of 5, but whi h don't in lude 5 itself? If it is possible, write
down su h a set. If it is not possible, explain brie y why it annot be
done.
Solution by Alex Song, student, Elizabeth Ziegler Publi S hool, Waterloo,
ON.
(a) The set {5, 6, 7, 8, 9} works.
(b) The sets {2, 4, 9}, {2, 6, 7}, {3, 4, 8}, {3, 8, 9}, and {4, 7, 9} all work.
( ) Let Dn denote the set of allowed digits that leave remainder n when
divided by 5, 0 < n < 5. Then D1 = {6}, D2 = {2, 7}, D3 = {3, 8}, and
D4 = {4, 9}. Now two possibilities exist: either use one digit from ea h of
D1 , D2 , D3 , and D4 , or use two digits from ea h of D2 and D3 . The rst
ase leads to 1 · 2 · 2 · 2, or eight sets, while the se ond leads to one set. Thus
there are a total of nine sets.
(d) Any of the sets {1, 2, 3, 4, 6, 9}, {1, 2, 3, 4, 7, 8}, {1, 2, 3, 7, 8, 9},
{1, 2, 4, 6, 8, 9}, {1, 3, 4, 6, 7, 9}, {1, 4, 6, 7, 8, 9}, {2, 3, 4, 6, 7, 8},
or {2, 3, 6, 7, 8, 9} will work.
(e) The eight allowed digits 1, 2, 3, 4, 6, 7, 8, and 9 add up to 40. If the sum
of seven of these were divisible by 5, then the remaining digit would also be
divisible by 5. This is not the ase, so no su h set of seven digits exists.
Also solved by JOCHEM VAN GAALEN, student, Medway High S hool, Arva, ON; and
JIXUAN WANG, student, Don Mills Collegiate Institute, Toronto, ON.
In Part (a), Jixuan Wang points out that the largest possible sum of ve distin t digits is
5 + 6 + 7 + 8 + 9 = 35, so {5, 6, 7, 8, 9} is the only solution.
In Part (b), nding a set is suÆ ient; to see that these are the only possible solutions,
systemati ally onsider all sets of three digits. You an begin by onsidering sets ontaining the
digit 2, then sets whose smallest member is 3, and so on.
In Part ( ), 1 + 2 + 3 + 4 = 10 is divisible by 5, so hoosing one number from ea h
set works. Now, put one number ba k into the set it ame from and repla e it by a number
from a di erent set. This auses the sum 10 to hange by ±1, ±2, or ±3, and no new sum is
divisible by 5. Therefore, you annot pi k two numbers from one set and one number from ea h
of two di erent sets. That only leaves the possibility of hoosing the four numbers in D2 ∪ D3 ,
D2 ∪ D4 , or D3 ∪ D4 , and of these possibilities only D2 ∪ D3 yields a solution.
358

In Part (d), nding a set is suÆ ient; to see that these are all of the solutions, note that
the sum of the eight allowed digits is 40 and that, if six hosen digits have a sum divisible by 5,
then the remaining two digits also have a sum divisible by 5. Systemati ally sear hing for all
su h sets of size two yields {1, 4}, {1, 9}, {2, 3}, {2, 8}, {3, 7}, {4, 6}, {6, 9}, and {7, 8}.
The omplements of these sets are the sets given in the solution above (in reverse order).

2 . Braille is a ode whi h lets blind people read and write. It was invented
by a blind Fren hman, Louis Braille, in 1829. Braille is based on a pattern
of dots embossed on a 3 by 2 re tangle. It is read with the ngers moving
a ross the top of the dots.
Altogether there are 63 possible ways to emboss one to six dots on a 3
by 2 re tangle. (We will not ount zero dots in this question.)
Figure 1 shows the pattern for the letter h. Note that if we re e t this
pattern (down the middle of the re tangle) the result is the Braille letter j,
shown in Figure 2.
................................................................
...................................................................... .... ...
....
.... } ...
....
...
....
.... } ...
.....
... ... ...
.... .... .... .... ....................................................................
...
}
.... .... ... ... ...
} }
... ....
} }
... .... ... ....
... ...
.... . ... .... .... ...
... ..... .... ... .... ...
...
... .. .... ... ...
.... .... .... ... ... ...
}
...
.... .... ... .... .... .
.... ... .... ... .... .....
... .... .... .... ... ...
.... ... ... .. ...................................................................
................................................................. ................................................................

Fig. 1: Fig. 2: Fig. 3: a simpli ed


the letter h the letter j version of Braille.
(a) How many di erent patterns are possible using just one dot?
(b) There are 15 di erent ways to emboss two dots on a 3 by 2 re tangle.
How many ways are there to emboss four dots on a 3 by 2 re tangle?
Brie y explain your answer.
( ) In luding the two patterns shown in Figures 1 and 2, how many possible
patterns are there using three dots?
(d) A simpli ed version of Braille has been proposed. In this version the
dots will be embossed on a 2 by 2 re tangle. An example is shown in
Figure 3. How many possible patterns would there be in this simpli ed
version (assuming that we will not ount zero dots)?
(e) Write down how many possible patterns there will be if we were to
develop a more ompli ated version of Braille using a 4 by 2 re tangle.
(Again assume that we would not ount zero dots.)
Solution by Jixuan Wang, student, Don Mills Collegiate Institute, Toronto,
ON.
(a) The single dot must be pla ed in one of the six positions of a two-by-
three re tangle, so six patterns exist.
(b) There are fteen ways. For every two-dot pattern, swapping dots for
blanks and blanks for dots produ es a unique four-dot pattern. Therefore
equally many two-dot and four-dot patterns exist.
359

Alternatively, the number of ways one an hoose four positions from


the six possible positions is 6 C4 = 64 = 4! (66!− 4)! = 15. Of ourse 62 is
 

also 15.
( ) If you remove one dot from a four-dot pattern, a three-dot pattern re-
mains. Sin e any of the four dots an be removed, ea h four-dot pattern gives
rise to four three-dot patterns in this way. However, ea h possible three-dot
pattern arises from three di erent four-dot patterns, namely one four-dot
pattern for ea h of the three blanks in the three-dot pattern. Sin e the num-
ber of four-dot patterns is fteen, it follows that the number of three-dot
patterns is 153· 4 = 20.
Alternatively, the number of ways one an hoose three positions from
the six possible is 6 C3 = 63 = 3! (66!− 3)! = 20.


(d) Here ea h pattern has four positions. Thus you an arrange one dot in
four ways, three dots (so one blank) in four ways, and four dots in one way.
By reasoning as in Part ( ), the number of two-dot patterns is 4 · 3/2 = 6.
Thus the total number of patterns  is 4 + 4 + 1 + 6 = 15.
Alternatively, 41 + 42 + 43 + 44 = 4 + 6 + 4 + 1 = 15 is a di erent


way to ount the number of patterns.


(e) Here ea h pattern has eight positions, so the number of patterns is
8
1
+ 8
2
+ · · · + 8
8
= 8 + 28 + 56 + 70 + 56 + 28 + 8 + 1 = 255.
Also solved by JOCHEM VAN GAALEN, student, Medway High S hool, Arva, ON; and
ALEX SONG, student, Elizabeth Ziegler Publi S hool, Waterloo, ON.
In Parts (b) through (d) van Gaalen ommendably provides solutions from rst prin iples
in the spirit of the question rather than resorting to general ounting methods. However, as
his alternative solutions show, familiarity with binomial oeÆ ients simpli es the solutions
onsiderably.
Song provides even simpler solutions to Parts (d) and (e): Ea h of the four (respe tively
eight) positions must be either blank or a dot. Thus 24 (respe tively 28 ) patterns exist. Disal-
lowing the all-blank pattern thus leaves fteen (respe tively 255) patterns.

3. Around the year 2000 BC, the Babylonians used a number system based
on the number 60. For example, where we would write 0.25 (meaning \one
quarter" or 1/4), they would write something like || 15 | (meaning \ fteen
sixtieths" or 15/60, whi h does simplify to be ome 1/4 in our number sys-
tem).
The table below shows some numbers and their re ipro als written a -
ording to the Babylonian system. (A spe ial feature of a number and its
re ipro al is that when you multiply them together, the result is always 1.)
In the table below, ea h olumn represents a pla e value of 1/60 of the pre-
vious olumn. For example, 7 | 30 means 7/60 + 30/3600.
Number || Re ipro al Number || Re ipro al
2 || 30 | 6 || |
3 || 20 | 7 || | | | | ...
4 || | 8 || 7 | 30 |
5 || | 9 || | |
360

(a) Write down in the orre t order the three missing numbers whi h should
go beside 4, 5, and 6.
(b) If we added our number 2.5 (meaning \two and a half") into the table
above, what number would we write beside it to show the Babylonian
version of its re ipro al?
( ) The re ipro al of the number 8 is shown in the table as 7 | 30 |. Using
the same notation, what is the re ipro al of the number 9?
(d) Write down the rst two numbers whi h should go beside 7 in the table
above.
(e) Write down the Babylonian version for the re ipro al of our number
192.

Solution by Jixuan Wang, student, Don Mills Collegiate Institute, Toronto,


ON.
(a) Sin e 14 = 15 , 1 = 12
60 5 60
, and 16 = 6010
; the re ipro al of 4 is k15|, the
re ipro al of 5 is k12|, and the re ipro al of 6 is k10|.
1 1 2 24
(b) Sin e 2.5 =
5/2
= =
5 60
, the re ipro al of 2.5 is k24|.
( ) Sin e 19 = 400
602
=
360 + 40
602
=
6
60
40
+ 2 , the re ipro al of 9 is k6|40|.
60
x 1
(d) The largest integer, x, su h that 60 <
7
is 8. Thus, the rst digit is
1 8 1
x = 8. Now −
7 60
=
105
1
, and the largest integer, y, su h that 60y 2 < 105
is 34. Thus the se ond digit is y = 34. Therefore, the re ipro al of 7 begins
k8|34| . . . .
1 1125 1080 + 45 18 45
(e) Sin e 192 =
603
=
603
=
602
+
603
, the re ipro al of 192
is k0|18|45|.
Also solved by ALEX SONG, student, Elizabeth Ziegler Publi S hool, Waterloo, ON; and
(in part) JOCHEM VAN GAALEN, student, Medway High S hool, Arva, ON.
You may enjoy verifying that the re ipro al of 7 (see Part (d)) ontinues the pattern
k8|34|17|8|34|17| . . . . That is, the Babylonian \digits" 8|34|17| ontinue inde nitely.

4 . When we nally landed on Mars, we dis overed that Martians love to


play a game alled HitBall. In this game two teams of players try to hit a ball
between poles pla ed at ea h end of a eld. The team that s ores the most
points within one Martian hour is the winner.
There are three ways to s ore points. An Inner s ores 7 points, an Outer
s ores 4 points, while a Wide s ores 2 points.
The rst mat h report sent ba k to Earth was not very lear be ause of
stati , so not all the details are ertain. However, we did hear that the Red
Team won. They s ored \something{seven" points altogether (only the last
number ould be learly heard). We also learned that they had 16 su essful
s oring shots.
361

(a) Write down in a list (from smallest to largest) the possible numbers
of Inners whi h the Red Team ould have s ored a ording to this rst
mat h report.
(b) A later report added the information that the Red Team s ored the same
number of Inners and Outers. Using this extra information, write down
how many points the Red Team s ored altogether.
( ) Explain why your answer to (b) is the only possible solution. As part
of your explanation, make sure you in lude how many Inners, Outers,
and Wides the Red Team s ored.

Solution by Alex Song, student, Elizabeth Ziegler Publi S hool, Waterloo,


ON.
(a) The Red Team must have s ored an odd number of Inners, sin e the
other point s ores are even and the total number of points won by the Red
Team is odd. Thus, the number of Inners must be one of 1, 3, 5, 7, 9, 11, 13,
or 15.
Suppose the Red Team s ored x Inners, 0 Outers, and 16 − x Wides.
Then the total number of point is 2(16 − x) + 7x = 32 + 5x. Sin e x is odd,
32 + 5x is \something-seven." Thus any of 1, 3, 5, 7, 9, 11, 13, or 15 are
possible.
(b), ( ) Again let x be the number of Inners. Then the Red Team s ored
x Outers and 16 − 2x Wides, and 7x + 4x + 2(16 − 2x) = 7x + 32 is the
total number of points. If 7x + 32 leaves remainder 7 when divided by 10,
then 7x + 25 is divisible by 10, as is 7x + 5. Therefore 7x must be an odd
multiple of 5, so x must be an odd multiple of 5. Sin e the number of Wides,
16 − 2x, must be nonnegative, it follows that x = 5 and that this is the only
solution. The total number of points is then 7 · 5 + 32 = 67.
Also solved by JIXUAN WANG, student, Don Mills Collegiate Institute, Toronto, ON;
and JOCHEM VAN GAALEN, student, Medway High S hool, Arva, ON.
Our solver's original solution used modular arithmeti : If the integers x and y leave
the same remainder when divided by the natural number n, then we say that x and y are
equivalent modulo n, and we write x ≡ y (mod n). In Part (b), 7x + 32 ≡ 7 (mod 10), so
7x + 25 ≡ 7x + 5 ≡ 0 (mod 10). Thus 7x ≡ 5 (mod 10), when e x ≡ 5 (mod 10). That
is, x is an odd multiple of 5.

5 . During 2004, a Dunedin newspaper held a ompetition to nd a new ag


design for the provin e of Otago. Wendy entered the ompetition. Her entry
was based on the design shown below (Figure 4). Her ag featured a gold
ross with a blue ba kground. She also pla ed a ir le into her design. The
top and bottom of the ir le just tou h the orners of the top and bottom
triangles, as shown in Figure 4.
(a) Wendy designed her ag to be 240 m long and 150 m high. If the
edges of the ross are 40 m and 30 m away from ea h of the orners,
as shown in Figure 4, what is the radius of the entre ir le?
362

(b) Wendy de ided to remove the ir le from her design (see Figure 5). With
the ir le removed, what is the total area of the ross?
40
- ... ... ...
40
- ...
...... .................................................................................................................................................................................................... ................................................................................................................................................................................................

30 6 6
... ....... .. .... ... ....... .... ...
...... ......
30
....... .... .......
.... ..
.......
? ?
....... ..... .
. ....... ....
... ....... ....... ..... .... ....... ....... ...
...... ...........
....... ...... ....... ......
....... ....... ..........
........ ........... ....... ........ . ..
.... ....
. .
... ......... ....... ....... ....... ..... .... .......... ....... ..
.. .
..
.
.... .......... ....... ...
. ........ ...... .... ....... .......
. . .
..
.....
. . .
.
. .
..
.
..
.... ...
....... .... .... ....... ....... ...... ..... .
.... ....... . .
... . ...... .
.....
. .
..
. ..... .... ....... .............
.....
.
. .
.. .....
....
....... .......................... ....... .... ....... ...... ....
....... ....... .... .......
... ....... .......
.... .......
....... .... .. ...... .... .... ....... ....... ...
... ....... .... ...
..... ..... .... .... .......
...... ....... ..... ....
.... ...... ... .
. ..
... ... .... . ... ....
.... ... ..... ... .. .........
. .... .... .......... .......
.. ....
...... .. .. ....... ...... .......
.... .... ...... .. ..
........ .... .... ... . ... ....... ....
.... .......... ... .................... .....
... .. .
. .... .... .......... .. ......... ...
.... .
.... ..... ..
. . . ...
.......
..
........ .. ... . ..
. .. .... ..
.................
. ....... ..
........ ...
... .
..
......
. . ..
. ...... ........... .
........ .....
. .... ............
...
.......... ..
........ ..
. ....... .....
... ........... ....
...... .......
......... ................ .............. ....
. ..... ..
.. ..
............
...... ..........
. ..
. ...
. ..... ..
.. .. ........ .... .......... .......
. .. ..

30 6 6
..... ....... ... .......
...... ....... .... ... ....... ....... ....
30
...
....... ....... ... ....... .......
? - ?
.... ...... ....... ... ....... ....... ....
. ..... .
-
... .... . ... ... ..
. .. .
.. ..
...
...... ..................................................................................................................................................................................................
... .........................................................................................................................................................................................
... ... ... ...
. . . .
40 40
Fig. 5 (not to s ale)
Fig. 4 (not to s ale)
Solution by Alex Song, student, Elizabeth Ziegler Publi S hool, Waterloo,
ON.
(a) Label the points as shown.
Sin e it is a ross, K , P , M , and A B C
C are ollinear. Thus, △QP M ∼ ...................................................................................................................... D
△RP K ∼ △ACK , and we have
....... . ..
...
..... ....... ......... .
L .. ...... .... E
...
QM RK AK 150 − 30 3
....... ..
= . .......M...... ... ...
= = = ... ....... .. ..
.... .
QP RP AC 240 − 40 5 .... ...... ...............
. . . ...... ... ....

Now R must be the midpoint of AJ , ... .......... .. .... ...


... ..... .... .... ...
.... ...
..... .. .. ...... .. ...... ...
R .... N
..
...........................................................
150
so RK = 2 − 30 = 45. Thus,
..
... P ..... Q... .........
... . .. .
...
... ..... ..
... .......... ................. ......
....... ...
5
RP = (45) = 75, and moreover
.
.. ........ . .
....... O ............
.
. ..
. ..
. .
..... . .........
3
........
1 K . .. F
RP + P Q = AD = 120, so that
.
.
... .. ..
. . .......
.
2 ......................................................................................................................
..... .
3
P Q = 45. Hen e QM = QP = 27. J I H G
5
That is, the radius is 27 m.
(b) Sin e QM + M S = 12 AJ = 75, it follows that M S = 48, when e the
area of △BM C is 1602· 48 = 3840. Likewise, the area of △HIO is 3840.
Moreover, the area of △LP K is 90 2· 75 = 3375, and the area of △EN F is
also 3375. Sin e the entire re tangle ADGJ has area 36000, the area of the
ross is 36000 − 2 · 3840 − 2 · 3375 = 21570 m2 .
Also solved by JIXUAN WANG, student, Don Mills Collegiate Institute, Toronto, ON.
Here is an analyti al approa h: Say J = (0, 0), A = (0, 150), D = (240, 150),
and G = (240, 0). Then K = (0, 30) and C = (200, 150), and the line CK is given by
y = 35
x + 30. If x = 120, then y = 3 5
· 120 + 30 = 102, so M = (120, 102). Sin e
Q = (120, 75), the radius is 102 − 75 = 27. Similarly, if y = 75, then 75 = 3 5
x + 30, so
x = 75, when e P = (75, 75). The area of the ross is now al ulated as above.

That ompletes another Skoliad. This issue's prize of a opy of CRUX


with Mayhem goes to Alex Song, student, Elizabeth Ziegler Publi S hool,
Waterloo, ON. We hope to re eive more solutions from more readers.
363

MATHEMATICAL MAYHEM
Mathemati al Mayhem began in 1988 as a Mathemati al Journal for and by
High S hool and University Students. It ontinues, with the same emphasis,
as an integral part of Crux Mathemati orum with Mathemati al Mayhem.
The Mayhem Editor is Ian VanderBurgh (University of Waterloo). The
other sta members are Monika Khbeis (As ension of Our Lord Se ondary
S hool, Mississauga) and Eri Robert (Leo Hayes High S hool, Frederi ton).

Mayhem Problems
Veuillez nous transmettre vos solutions aux problemes du present numero
avant le 15 de embre
 2009 . Les solutions re ues apres ette date ne seront prises
en ompte que s'il nous reste du temps avant la publi ation des solutions.
Chaque probleme sera publie dans les deux langues oÆ ielles du Canada
(anglais et fran ais). Dans les numeros 1, 3, 5 et 7, l'anglais pre edera le fran ais,
et dans les numeros 2, 4, 6 et 8, le fran ais pre edera l'anglais.
La reda tion souhaite remer ier Jean-Mar Terrier, de l'Universite de
Montreal, d'avoir traduit les problemes.

M381 . Corre tion. Propose par Mihaly Ben ze, Brasov, Roumanie.
Determiner
 toutes les solutions de l'equation

1 2 6 7
+ + + = x2 − 4x − 4 .
x−1 x−2 x−6 x−7

M401 
. Propose par l'Equipe de Mayhem.
Graham et Vazz sont en train de projeter une nouvelle pelouse aux
Quartiers Gen  eraux
 de CRUX. Graham dit : \ Si tu hoisis une pelouse 9
metres
 plus longue et 8 metres
 plus etroite,
 la surfa e sera la m^eme." Vazz
repond
 : \Si tu la hoisis 12 metres
 plus ourte et 16 metres
 plus large, la
surfa e sera aussi la m^eme." Quelles sont les dimensions de la pelouse pro-
jetee ?
M402 
. Propose par Ne ulai Stan iu, E ole Te hnique Superieure
 de Saint
Mu eni Sava, Ber a, Roumanie.
Trouver toutes les paires ordonnees
 (a, b) d'entiers tels que
ab ba + ab + ba = 89 .
364

M403 . Propose par Matthew Babbitt, etudiant,


 Albany Area Math Cir le,

Fort Edward, NY, E-U.
Jean a e rit
 un programme sur son ordinateur pour tester si un entier
plus grand que 1 est un nombre premier. Sa sournoise de soeur, Ali e, a edit
 e
le ode de telle sorte que si l'entree
 est impaire, la probabilite que le pro-
gramme donne une reponse
 orre te est de 52% et si l'entree
 est paire, ette
probabilite est de 98%. Jean veri e
 le programme en testant deux entiers
plus grands que 1 hoisis au hasard. Quelle est la probabilite que les deux
reponses
 soient orre tes ?
M404 . Propose par Bill Sands, Universite de Calgary, Calgary, AB.
Un magasin vend des opies d'un ertain arti le a x$ pie e,
 ou y$ pour a
opies, ou en ore z$ pour b opies, a et b etant
 des entiers tels que 1 < a < b
et x, y et z des nombres reels  positifs. Pour rendre le rabais \y$ pour a
opies" interessant,
 y$ devrait e^ tre plus bas que le prix paye pour a a hats
au prix de x$, don y < ax. Pour rendre le deuxieme  rabais \ z$ pour b
opies" interessant
 aussi, on pourrait insister sur une des deux onditions :
(a) zb < ay ; .-a-d.
 que le prix moyen pour une opie soit moindre que le
prix orrespondant du premier rabais.
(b) Chaque fois qu'on peut e rire
 b = qa + r ave q et r des entiers non
negatifs,
 alors on a z < qy + rx ; .-a-d.
 qu'il en outerait
^ plus her
d'a heter b opies en ombinant le premier rabais ave l'a hat individuel
de opies supplementaires
 plutot
^ que d'opter pour le deuxieme
 rabais.
Montrer que si la ondition (a) est satisfaite, alors la ondition (b) l'est aussi.
Donner aussi un exemple pour montrer que la ondition (b) pourrait e^ tre
satisfaite sans que la ondition (a) le soit.
M405 . Propose par George Apostolopoulos, Messolonghi, Gre e.

Trouver une formule donnant la valeur de la somme
17 + 187 + 1887 + 18887 + · · · + 188 . . . 87 ,

ou le dernier terme ontient exa tement n hi res 8.


M406 . Propose par Constantino Ligouras, etudiant,
 College
 S ienti que
E. Majorana, Putignano, Italie.
Le arre ABCD est ins rit dans le huitieme d'un er le de rayon 1 et
de entre O de sorte qu'il ait un sommet sur haque rayon et les sommets
B et C sur l'ar . Le arre EF GH est ins rit dans le triangle DOA de sorte
que E et H soient sur les rayons et F et G soient sur AD. Dans le probleme 
M295 [2007 : 200,
√ 202 ; solution 2008 : 203-204℄, on a vu que l'aire du arr e
2− 2
ABCD est . Trouver l'aire du arre EF GH .
3
.................................................................
365

M381 . Corre tion. Proposed by Mihaly Ben ze, Brasov, Romania.


Determine all of the solutions to the equation
1 2 6 7
+ + + = x2 − 4x − 4 .
x−1 x−2 x−6 x−7

M401 . Proposed by the Mayhem Sta .


Graham and Vazz were marking out a new lawn at CRUX Headquarters.
Graham said: \If you make the lawn 9 metres longer and 8 metres narrower,
the area will be the same". Vazz said: \If you make it 12 metres shorter and
16 metres wider, the area will still be the same". What are the dimensions
of the lawn?
M402. Proposed by Ne ulai Stan iu, Saint Mu eni Sava Te hnologi al
High S hool, Ber a, Romania.
Determine all ordered pairs (a, b) of positive integers su h that
ab ba + ab + ba = 89 .

M403 . Proposed by Matthew Babbitt, student, Albany Area Math Cir le,
Fort Edward, NY, USA.
Jason wrote a omputer program that tests if an integer greater than 1
is prime. His devious sister Ali e has edited the ode so that if the input is
odd, the probability that the program gives the orre t output is 52% and if
the input is even, the probability that the program gives the orre t output is
98%. Jason tests the program by inputting two random integers ea h greater
than 1. What is the probability that both outputs are orre t?
M404 . Proposed by Bill Sands, University of Calgary, Calgary, AB.
A store sells opies of a ertain item at $x ea h, or at a items for $y, or
at b items for $z , where a and b are positive integers satisfying 1 < a < b and
x, y , and z are positive real numbers. To make \a items for $y " a sensible
bargain, $y should be less than buying a separate items; in other words we
should have y < ax. To make \b items for $z " also a sensible bargain, we
ould insist on one of two onditions:
z y
(a) b
< ;
a
that is, the average pri e of an item under the \b items for $z "
deal is less than under the \a items for $y" deal.
(b) Whenever we an write b = qa + r for nonnegative integers q and r,
then z < qy + rx holds; that is, it should always ost more to buy b
items by buying a ombination of a items plus individual items, than
by hoosing the \b items for $z " deal.
Show that if ondition (a) is true, then ondition (b) is also true. Give an
example to show that ondition (b) ould be true while ondition (a) is false.
366

M405 . Proposed by George Apostolopoulos, Messolonghi, Gree e.


Determine a losed form expression for the sum
17 + 187 + 1887 + 18887 + · · · + 188 . . . 87 ,

where the last term ontains exa tly n 8's.


M406 . Proposed by Constantino Ligouras, student, E. Majorana S ien-
ti High S hool, Putignano, Italy.
Square ABCD is ins ribed in one-eighth of a ir le of radius 1 and
entre O so that there is one vertex on ea h radius and two verti es B and
C on the ar . Square EF GH is ins ribed in △DOA so that E and H lie
on the radii, and F and G lie on AD. In problem M295 [2007 : 200, 202; √
2− 2
solution 2008 : 203-204℄, we saw that the area of square ABCD is 3 .
Determine the area of square EF GH .

Mayhem Solutions
M369 . Proposed by the Mayhem Sta .
A re tangle has verti es A(0, 0), B(6, 0), C(6, 4), and D(0, 4). A hor-
izontal line is drawn through P (4, 3), meeting BC at M and AD at N . A
verti al line is drawn through P , meeting AB at Q and CD at R. Prove that
AP , DM , and BR all pass through the same point.

Solution by Jixuan Wang, student, Don Mills Collegiate Institute, Toronto,


ON, modi ed by the editor.
Sin e the sides of re tangle ABCD are parallel to the axes, any point
on BC has x- oordinate 6 and any point on CD has y- oordinate 4. Thus,
M has oordinates (6, 3) and R has oordinates (4, 4).
The line through A(0, 0) and P (4, 3) has slope 34 −
−0
0
=
3
4
and passes
3
through the origin, so has equation y = 4
x.
y−4 4−3
The line through D(0, 4) and M (6, 3) has equation x−0
=
0−6
, or
equivalently y = − 61 x + 4.
The line through B(6, 0) and R(4, 4) has equation xy −−6
0
=
0−4
6−4
, or
equivalently y = −2x + 12.
Next, we nd the point of interse tion of the lines AP and DM by
equating to obtain 34 x = − 16 x + 4, or 11
12
x = 4, or x =
48
11
. Sin e y = 34 x
367

3 48 36
on line AP , then y . Therefore, the point of interse tion of
 
= =
4 11  11
48 36
lines AP and DM is ,
11 11
.
Next, we nd the point of interse tion of lines AP and BR by equating
to obtain 34 x = −2x + 12, or 11 4
x = 12, or x =
48
. As before, we see that
11
48 36
the point of interse tion of lines AP and BR is 11 , 11 .


Sin e line AP interse ts lines DM and BR  at thesame point, it follows


that all three lines are on urrent, meeting at 48 ,
11 11
36
.
Also solved by EDIN AJANOVIC, student, First Bosniak High S hool, Sarajevo, Bosnia
and Herzegovina; GEORGE APOSTOLOPOULOS, Messolonghi, Gree e; JACLYN CHANG, stu-
dent, Western Canada High S hool, Calgary, AB; JULIA CLINE, student, Walt Whitman High
S hool, Bethesda, MD, USA; KATHERINE JANELL EYRE, student, Angelo State University, San
Angelo, TX, USA; ANTONIO GODOY TOHARIA, Madrid, Spain; RICHARD I. HESS, Ran ho

Palos Verdes, CA, USA; HUGO LUYO SANCHEZ, Ponti ia Universidad Catoli a
 del Peru, Lima,
Peru; RICARD PEIRO,  IES \Abastos", Valen ia, Spain; BRUCE SHAWYER, Memorial University
of Newfoundland, St. John's, NL; and NECULAI STANCIU, Saint Mu eni Sava Te hnologi al
High S hool, Ber a, Romania.
After nding the rst point of interse tion, it would suÆ e to show that this point lies
on line BR. Stri tly speaking, we have proven that the lines through A and P , D and M , and
B and R all pass through the same point; line segments DM and BR pass through this point,
but line segment AP does not ontain the point 48 , 36 (though its extension does).

11 11

M370 . Proposed by the Mayhem Sta .

(a) Prove that cos(A + B) + cos(A − B) = 2 cos A cos B for all angles
A and B .
C +D C−D
(b) Prove that cos C + cos D = 2 cos for all angles
   
cos
2 2
C and D .

( ) Determine the exa t value of cos 20◦ + cos 60◦ + cos 100◦ + cos 140◦ ,
without using a al ulator.

Solution by Courtis G. Chryssostomos, Larissa, Gree e.


(a) Applying the sum and di eren e formulae for the osine, we obtain
cos(A + B) + cos(A − B)
= (cos A cos B − sin A sin B) + (cos A cos B + sin A sin B)
= 2 cos A cos B .
C +D C−D
(b) Applying the formula from (a) with A = 2
and B =
2
, we
obtain A + B = C and A − B = D, and so
C +D C−D
.
   
cos C + cos D = 2 cos cos
2 2
368

( ) We rearrange the given expression and apply the formula from (b):
cos 20◦ + cos 60◦ + cos 100◦ + cos 140◦
= cos 140◦ + cos 20◦ + cos 100◦ + cos 60◦
   
140◦ + 20◦ 140◦ − 20◦
= 2 cos cos + cos 100◦ + cos 60◦
2 2
= 2 cos 80 cos 60 + cos 100 + cos 60◦
◦ ◦ ◦

1 1 1
 
= 2 cos 80◦ + cos 100◦ + = cos 80◦ + cos 100◦ +
2 2 2
1 1
= cos 80◦ + − cos 80◦ + .

=
2 2
Also solved by EDIN AJANOVIC, student, First Bosniak High S hool, Sarajevo,
Bosnia and Herzegovina; GEORGE APOSTOLOPOULOS, Messolonghi, Gree e; CAO MINH
QUANG, Nguyen Binh Khiem High S hool, Vinh Long, Vietnam; JULIA CLINE, student, Walt
Whitman High S hool, Bethesda, MD, USA; ANTONIO GODOY TOHARIA, Madrid, Spain;

RALPH LOZANO, student, Missouri State University, Missouri, USA; HUGO LUYO SANCHEZ,
Ponti ia Universidad Catoli a
  IES \Abastos", Valen ia,
del Peru, Lima, Peru; RICARD PEIRO,
Spain; NECULAI STANCIU, Saint Mu eni Sava Te hnologi al High S hool, Ber a, Romania;
VASILE TEODOROVICI, Toronto, ON; and JIXUAN WANG, student, Don Mills Collegiate
Institute, Toronto, ON. There were two in omplete solutions submitted.

M371 . Proposed by Panagiote Ligouras, Leonardo da Vin i High S hool,


No i, Italy.
Suppose that the line segment AB has length 3 and C is on AB with
AC = 2. Equilateral triangles ACF and CBE are onstru ted on the same
side of AB . If K is the midpoint of F C , determine the area of △AKE .
Solution by Antonio Godoy Toharia, Madrid, Spain.
Sin e K is the midpoint of segment F
1
F C , we have KC = (F C) = 1 = CE . ........
2
Be ause △ACF and △CBE are equilat-
.
... ......
.
.
eral, ∠F CA = ∠ECB = 60◦ , so we have
...
. ...
..
... K
... E
∠KCE = 60 .
. ... . ... ... ... ... ... ................
◦ .... ... .. ....... .. ..
... . ... ...
.
... . ... . ... ...
. ... ......
Therefore △KCE is also equilat-
.... ... ..... ..
. . ... ... ...
... ... .
... ..
. .. ..
. ...
.... ...
. ... ... ..
... .. . ...

eral, and so KE = 1.
.. .. ... ...
..... ...
. ....... ... .
..................................................................... ...................................
.
. ....... ..
Sin e ∠EKC = ∠KCA, then KE A C B
is parallel to AB , and so we an think
of △AKE as having base KE = 1 and height equal to the verti al distan e
between the lines through KE and AB . √
The height of △CBE is h = 1 sin 60 = 23 . Thus, the distan e



3
between KE and AB is 2
.
√  √
1 3 3
Therefore, the area of △AKE is equal to 2
(1)
2
=
4
.
Also solved by EDIN AJANOVIC, student, First Bosniak High S hool, Sarajevo, Bosnia
and Herzegovina; GEORGE APOSTOLOPOULOS, Messolonghi, Gree e; CAO MINH QUANG,
369

Nguyen Binh Khiem High S hool, Vinh Long, Vietnam; JACLYN CHANG, student, Western
Canada High S hool, Calgary, AB; JULIA CLINE, student, Walt Whitman High S hool, Bethesda,
MD, USA; EMILY HENDRYX, student, Angelo State University, San Angelo, TX, USA; JOSE

HERNANDEZ SANTIAGO, student, Universidad Te nologi a
 de la Mixte a, Oaxa a, Mexi o;
RICHARD I. HESS, Ran ho Palos Verdes, CA, USA; HUGO LUYO SANCHEZ,  Ponti ia
Universidad Catoli a
 del Peru, Lima, Peru; RICARD PEIRO, IES \Abastos", Valen ia, Spain;
and JIXUAN WANG, student, Don Mills Collegiate Institute, Toronto, ON. There were two
in orre t solutions submitted.

M372 . Proposed by the Mayhem Sta .


A real number x satis es x3 = x + 1. Determine integers a, b, and c
so that x7 = ax2 + bx + c.
Solution by Paul Bra ken, University of Texas, Edinburg, TX, USA.
Sin e x3 = x + 1, we have
x3 · x3 = (x + 1)(x + 1) = x2 + 2x + 1 ,
whi h yields
x7 x · x6 = x x2 + 2x + 1 = x3 + 2x2 + x

=
= (x + 1) + 2x2 + x = 2x2 + 2x + 1 .
Thus, if a = 2, b = 2, and c = 1, we have x7 = ax2 + bx + c.
Also solved by EDIN AJANOVIC, student, First Bosniak High S hool, Sarajevo, Bosnia
and Herzegovina; GEORGE APOSTOLOPOULOS, Messolonghi, Gree e; CAO MINH QUANG,
Nguyen Binh Khiem High S hool, Vinh Long, Vietnam; ANTONIO GODOY TOHARIA, Madrid,
Spain; JOSE HERNANDEZ
 SANTIAGO, student, Universidad Te nologi a
 de la Mixte a, Oaxa a,
Mexi o; RICHARD I. HESS, Ran ho Palos Verdes, CA, USA; HUGO LUYO SANCHEZ,  Ponti ia
Universidad Catoli a
 del Peru, Lima, Peru; RICARD PEIRO,  IES \Abastos", Valen ia, Spain;
NECULAI STANCIU, Saint Mu eni Sava Te hnologi al High S hool, Ber a, Romania; EDWARD
T.H. WANG, Wilfrid Laurier University, Waterloo, ON; and JIXUAN WANG, student, Don Mills
Collegiate Institute, Toronto, ON.
It an be proven using algebra that a = 2, b = 2, and c = 1 is the only solution.

M373 . Proposed by Kunal Singh, student, Kendriya Vidyalaya S hool,


Shillong, India.
The side lengths of a triangle are three onse utive positive integers
and the largest angle in the triangle is twi e the smallest one. Determine the
side lengths of the triangle.
I.Solution by Jixuan Wang, student, Don Mills Collegiate Institute, Toronto,
ON.
Suppose that the side lengths of the triangle are n − 1, n, and n + 1
for some positive integer n ≥ 3. (Smaller values of n do not give a triangle.)
Suppose that the smallest angle is θ, whi h is opposite the shortest side (of
length n − 1). Thus, the largest angle is 2θ, whi h is opposite the longest
side (of length n + 1).
By the Law of Sines, nsin
−1
θ
=
n+1
sin 2θ
and so nsin
−1
θ
=
n+1
2 sin θ cos θ
. Sin e
sin θ 6= 0, then cos θ = 2(n−1) .
n+1
370

Now, by the Law of Cosines,


(n − 1)2 = n2 + (n + 1)2 − 2n(n + 1) cos θ ;
n+1
(n − 1)2 = n2 + (n + 1)2 − 2n(n + 1) ;
2(n − 1)
n(n + 1)2
n2 − 2n + 1 = n2 + n2 + 2n + 1 − ;
n−1
n(n + 1)2
= n2 + 4n ;
n−1
2
n(n + 2n + 1) = (n2 + 4n)(n − 1) ;
n2 + 2n + 1 = (n + 4)(n − 1) (sin e n 6= 0) ;
n2 + 2n + 1 = n + 3n − 4 ;
2

n = 5.

Therefore, n = 5, and the side lengths are 4, 5, and 6.


II.Solution by Vasile Teodorovi i, Toronto, ON.
Let the triangle be ABC , with side lengths AB = m, AC = m + 1,
and BC = m + 2 for some positive integer m ≥ 2. Then the smallest angle
is ∠ACB , whi h we label θ, and the largest is ∠BAC , whi h we label 2θ.
Let AD be the angle bise tor of
∠BAC , with D on BC . Let BD = x and
DC = y . Note that BC = AB + 2, so A
x + y = m + 2. ..........
.. ... ......
.
... ... ..... ... ..... ........
m ....... ..... m + 1
By the Angle Bise tor Theorem,
... .....
. ... .....
.. .
...
... .....
AC DC
, so mm
+1
= .
y . ... .....
= .. . . .....
. ...
AB DB x ... ... .....
. .
...............................................................................................................
m+1 y B x y C
Thus, m
+1 =
x
+ 1 and so D
2m + 1 x+y m+2
m
=
x
=
x
.
This gives x = m(m + 2)
2m + 1
and y = mm+1
x=
(m + 1)(m + 2)
2m + 1
.

Next, we note that ∠BAD = ∠BCA = θ so △BAD is similar to


BD BA m(m + 2)  m
△BCA, when e = or m = , whi h gives the
BA BC 2m + 1 m+2
equivalent equations (m + 2) = m(2m + 1), and m + 4m + 4 = 2m2 + m,
2 2

and 0 = m2 − 3m − 4.
Sin e m is a positive integer, then m = 4, and so the side lengths are
4, 5, and 6.
Also solved by EDIN AJANOVIC, student, First Bosniak High S hool, Sarajevo, Bosnia
and Herzegovina; GEORGE APOSTOLOPOULOS, Messolonghi, Gree e; CAO MINH QUANG,
Nguyen Binh Khiem High S hool, Vinh Long, Vietnam; COURTIS G. CHRYSSOSTOMOS, Larissa,
Gree e; ANTONIO GODOY TOHARIA, Madrid, Spain; RICHARD I. HESS, Ran ho Palos Verdes,
371

CA, USA; MISSOURI STATE UNIVERSITY PROBLEM SOLVING GROUP, Spring eld, MO, USA;
RICARD PEIRO,  IES \Abastos", Valen ia, Spain; NECULAI STANCIU, Saint Mu eni Sava
Te hnologi al High S hool, Ber a, Romania; and EDWARD T.H. WANG, Wilfrid Laurier
University, Waterloo, ON.

M374 . Proposed by Mihaly Ben ze, Brasov, Romania.


Suppose that p is a xed prime number with p ≥ 3. Determine the
number of solutions to x3 + y3 = x2 y + xy2 + p2009 , where x and y are
integers.
Solution by Missouri State University Problem Solving Group, Spring eld,
MO, USA.
The equation an be rewritten as
p2009 = x3 − x2 y − xy 2 + y 3 = x2 (x − y) − y 2 (x − y)
= (x − y)(x2 − y 2 ) = (x − y)2 (x + y) .
Sin e x and y are integers and the only divisors of p2009 are of the form
±pn , then x − y = ±pk for some integer k with 0 ≤ k ≤ 1004. Thus,
(x − y)2 = p2k and so x + y = p2009−2k . (The upper bound on k omes from
the fa t that (x − y)2 is also a divisor of p2009 .)
This yields
1 1 2009−2k
± pk ,
 
x = (x + y) + (x − y) = p
2 2
1 1 2009−2k
∓ pk .
 
y = (x + y) − (x − y) = p
2 2
These are all integers sin e p is odd (in fa t this is still true if p = 2 as long
as k > 0). Ea h pair (x, y) is distin t sin e for ea h pair the values of x + y
and x − y are di erent.
Sin e there are 1005 possible values for k in the range 0 ≤ k ≤ 1004,
there are then 2(1005) = 2010 solutions.
Also solved by EDIN AJANOVIC, student, First Bosniak High S hool, Sarajevo, Bosnia
and Herzegovina; and EDWARD T.H. WANG, Wilfrid Laurier University, Waterloo, ON. There
were four in orre t solutions and one in omplete solution submitted.

M375. Proposed by Ne ulai Stan iu, Saint Mu eni Sava Te hnologi al


High S hool, Ber a, Romania.
Determine all real solutions to the system of equations
1 4 9 9
+ 2 + 2 = 4; x2 + y 2 + z 2 = 9 ; xyz = .
x2 y z 2

Solution by Edward T.H. Wang, Wilfrid Laurier University, Waterloo, ON.


√
√ 
6 √ 3 2
There are exa tly four solutions for (x, y, z), namely 2 , 3, 2 ,
√ √   √ √   √
√ 6 √ √ 3√2

6 3 2 3 2 6
2
, − 3, −
2
, − 2 , 3, − 2 , and − 2 , − 3, 2 .
372

Note that if (x, y, z) is a solution, then so are (x, −y, −z), (−x, y, −z),
and (−x, −y, z). (No other ombination of minus signs will work, sin e an
even number of minus signs is ne essary to make the sign of xyz orre t.)
Hen e, we may assume that x, y, z > 0 (sin e none of x, y, z an equal 0).
The Cau hy{S hwarz Inequality says that if a, b, c, A, B , and C are
real numbers, then
a2 + b2 + c2 A2 + B 2 + C 2 ≥ (aA + bB + cC)2 ,
 

with equality if and only if (a, b, c) is a s alar multiple of (A, B, C).


By the Cau hy{S hwarz Inequality, we have
1 4 9
 
x2 + y 2 + z 2

36 = + +
x2 y2 z2
 2
1 2 3
    
≥ x +y +z = 36 .
x y z
 
1 2 3
Sin e equality holds, then (x, y, z) must be a s alar multiple of , ,
x y z
.
Thus, x = xk , y = 2k
y
, and z = 3k
z
, for some k > 0, and so x2 = k, y2 = 2k,
and z 2 = 3k.
Substituting ba k into r
the original

se ond equation, we
r obtain√6k = 9,

and so k = 2 , when e x = 2 = 2 , y = 3, and z = 92 = 3 2 2 . This
3 3 6

gives the four solutions above, as laimed.


Also solved by GEORGE APOSTOLOPOULOS, Messolonghi, Gree e; CAO MINH QUANG,
Nguyen Binh Khiem High S hool, Vinh Long, Vietnam; COURTIS G. CHRYSSOSTOMOS, Larissa,
 IES \Abastos",
Gree e; RICHARD I. HESS, Ran ho Palos Verdes, CA, USA; and RICARD PEIRO,
Valen ia, Spain. There were two in omplete solutions submitted.

Problem of the Month


Ian VanderBurgh

Sometimes we mathemati ians like to use a sledgehammer when per-


haps a smaller instrument would be in order.
Problem 1 (2009 Gauss Contest, Grade 7) If x, y, and z are positive integers
with xy = 18, xz = 3, and yz = 6, what is the value of x + y + z ?
(A) 6 (B) 10 (C) 25 (D) 11 (E) 8
If we weren't in Grade 7, then here are two ways that we might ta kle
this problem:
373

Solution 1 Sin e xy = 18, xz = 3, and yz = 6, by multiplying these equa-


tions we have (xy)(xz)(yz) = 18(3)(6), or x2 y2 z 2 = 182 , or (xyz)2 = 182 .
Sin e x, y, and z are positive, then xyz > 0, so xyz = 18. We an
now ombine this with the original three equations as follows.
Sin e xy = 18, we have xyz
xy
=
18
18
, or z = 1. Sin e xz = 3, we have
xyz 18
xz
=
3
, or y = 6. Sin e yz = 6, we have xyz
yz
=
18
6
, or x = 3.
Therefore, x + y + z = 3 + 6 + 1 = 10.
Solution 2Sin e xy = 18 and xz = 3, then xyxz
=
18
3
, or yz = 6, or y = 6z .
Sin e yz = 6, then (6z)z = 6, or z 2 = 1. Sin e z > 0, then z = 1, and so
y = 6z = 6. Sin e xz = 3 and z = 1, then x = 3.
Therefore, x + y + z = 3 + 6 + 1 = 10.
These are two great solutions using standard te hniques for solving sys-
tems of equations. But they're hardly suitable for Grade 7 students. This
problem is a perfe tly good Grade 7 problem, though.
Solution 3 We are told that x, y , and z are positive integers. Sin e xz = 3
and 3 is a prime, then x and z must be 1 and 3, or 3 and 1, respe tively.
Let's look at the ase x = 1 and z = 3. Sin e x = 1 and xy = 18
and the number that we multiply 1 by to get 18 is 18 itself, then y = 18.
Therefore, yz = 18(3) = 54, whi h disagrees with the equation yz = 6.
Thus, x = 1 and z = 3 is not the orre t ombination. (For the re ord, I
never hoose the orre t line in the gro ery store either.)
So we look at the ase x = 3 and z = 1. Sin e x = 3 and xy = 18 and
the number that we multiply 3 by to get 18 is 6, then y = 6. (The produ t
yz does in fa t equal 6, as required). Thus, x = 3, y = 6, and z = 1.
Therefore, x + y + z = 3 + 6 + 1 = 10.
It's easy to get \trapped" into using the high powered te hniques that
we know, but sometimes there is a ni er solution that uses less ma hinery.
Here is a problem in a similar vein that has developed a following in
the math ontest world.
Problem 2 (1988 UK S hools Mathemati al Challenge) Weighing the baby at
the lini was a problem. The baby would not keep still and aused the s ales
to wobble. So I held the baby and stood on the s ales while the nurse read
o 78 kg. Then the nurse held the baby while I read o 69 kg. Finally, I held
the nurse while the baby read o 137 kg. What is the ombined weight of all
three (in kg)?
(A) 142 (B) 147 (C) 206 (D) 215 (E) 284
Again, there is a standard system-of-equations type solution whi h is
worth seeing. To save onfusion, we'll give the narrator a name hosen at
random, say, Tony.
374

Solution 1 Let Tony's weight be x kg, let the baby's weight be y kg, and let
the nurse's weight be z kg.
The ombined weight of Tony and the baby is 78 kg, so x + y = 78.
The ombined weight of the baby and the nurse is 69 kg, so y + z = 69. The
ombined weight of Tony and the nurse is 137 kg, so x + z = 137.
We an show this system of equations ni ely in a visual way:
x + y = 78 ,
y + z = 69 ,
x + z = 137 .

By the way, laying out the equations in this fashion is a really useful thing to
do and gives you a mu h better idea of what to do than by writing
x + y = 78 ,
y + z = 69 ,
x + z = 137 .

In any event, the \ni e" way of writing the equations allows us to see that
adding the three equations is a really good idea. When we do this, we obtain
2x + 2y + 2z = 78 + 69 + 137, or 2(x + y + z) = 284, or x + y + z = 142.

Noti e that we don't a tually need to determine x, y, and z at all!


But, again, we don't need to do anything nearly that fan y. In fa t, we
an get away without doing any algebra at all.
Solution 2 First, we look at the fa t that the ombined weight of Tony and
the baby is 78 kg and the ombined weight of the nurse and the baby is 69 kg.
Sin e the baby's weight is in luded in both of these totals, then Tony must
be 78 − 69 = 9 kg heavier than the nurse (that is, the di eren e between
Tony's weight and the nurse's weight is 9 kg).
But the ombined weight of Tony and the nurse is 137 kg. We want to
nd two numbers that add to 137, one of whi h is 9 greater than the other.
To nd these numbers, we an subtra t 9 from 137 to get 128 and then divide
by 2 to get 64. The numbers 64 and 64 + 9 = 73 di er by 9 and add to 137,
so must be the weights, in kg, of the nurse and Tony, respe tively.
Sin e the ombined weight of Tony and the baby is 78 kg and Tony
weighs 73 kg, then the weight of the baby is 5 kg.
Therefore, the ombined weight of all three is 64 + 73 + 5 = 142 kg.
So put your sledgehammer ba k in the garage, and think before you
leap.
375

THE OLYMPIAD CORNER


No. 280

R.E. Woodrow

As a rst problem set for this number we look to the Grade 12 problems
of the Final Round of the Estonian National Olympiad 2005{2006. My thanks
go to Robert Morewood, Canadian Team Leader to the 2006 IMO in Slovenia,
for olle ting them for our use.
ESTONIAN NATIONAL OLYMPIAD 2005{2006
Final Round

Grade 12

1 . We all a gure a ship if it is made up of unit squares onne ted by om-


mon edges. Prove that if there is an odd number of possible di erent ships
onsisting of n unit squares on a 10 × 10 board, then n is divisible by 4.
2 . Find the smallest distan e between points P and Q in the xy-plane, if P
lies on the line y = x and Q lies on the urve y = 2x .
3 . Prove or disprove the following statements:
(a) For every integer n ≥ 3, there exist n distin t positive integers su h that
the produ t of any two of them is divisible by the sum of the remaining
n − 2 numbers.

(b) For some integer n ≥ 3, there exist n distin t positive integers su h


that the sum of any n − 2 of them is divisible by the produ t of the
remaining two numbers.

4. The a ute triangle ABC has ir um entre O and triangles BCO , CAO ,
and ABO have ir um entres A′ , B ′ , and C ′ , respe tively. Prove that the
area of triangle ABC does not ex eed the area of triangle A′ B ′ C ′ .
5 . The Ababi alphabet onsists of the letters A and B , and the words in the
Ababi language are pre isely those that an be formed by the following two
rules:
(a) The letter A is a word.
(b) If s is a word, then s ⊕ s and s ⊕ s̄ are words, where s̄ denotes the word
that is obtained by repla ing all letters A in s with letters B , and vi e
versa; and x ⊕ y denotes the on atenation of words x and y.
376

The Ululu alphabet also onsists of the letters A and B , and the words
in the Ululu language are pre isely those that an be formed by the following
two rules:
(a) The letter A is a word.
(b) If s is a word, then s⊗s and s⊗ s̄ are words, where s̄ is de ned as above
and x ⊗ y is the word obtained from words x and y of equal length by
writing the letters of x and y alternatingly, starting from the rst letter
of x.
Prove that the two languages onsist of the same words.

Next we turn to the Kazakhs and the problems of the II International


Zhautykov Olympiad in Mathemati s, Almaty, 16{17 January, 2006. Thanks
again go to Robert Morewood, Canadian Team Leader to the 2006 IMO in
Slovenia, for olle ting them for us.
II INTERNATIONAL ZHAUTYKOV OLYMPIAD
IN MATHEMATICS
Almaty { Day 1 (January 16, 2006)

1 . Find all positive integers n su h that n = ϕ(n)+402, where ϕ is the Euler


phi-fun tion (it is known
 that  if p1 , p2 , . 
. . , pkare the 
distin t prime divisors
1 1 1
of n, then ϕ(n) = n 1 − p 1−
p
··· 1 −
p
; and ϕ(1) = 1).
1 2 k

2 . The points K and L lie on the sides AB and AC , respe tively, of the
triangle ABC su h that BK = CL. Let P be the point of interse tion of
the segments BL and CK , and let M be an inner point of the segment AC
su h that the line M P is parallel to the bise tor of the angle ∠BAC . Prove
that CM = AB .
3. An m × n table with 4 ≤ m ≤ n is alled good if the numbers 0 and 1 an
be written in the unit squares of the table su h that the following onditions
hold:
(a) not all numbers are 0's and not all are 1's;
(b) the number of 1's in any 3 × 3 subsquare is onstant;
( ) the number of 1's in any 4 × 4 subsquare is also onstant.
Find all pairs of positive integers (m, n) for whi h there is a good m×n
table.
377

Almaty { Day 2 (January 17, 2006)

4 . A heap of 100 stones is given. A partition of the heap into k smaller heaps
is spe ial provided that (i) no two heaps have the same number of stones,
and (ii) any further partition of any of the heaps into two new heaps results
in k + 1 heaps for whi h (i) fails (a heap always ontains at least one stone).
(a) Find the maximum number k for whi h there is a spe ial partition of
the given heap of 100 stones into k heaps.
(b) Find the minimum number k for whi h there is a spe ial partition of the
given heap of 100 stones into k heaps.

5 . Let a, b, c, and d be real numbers su h that a + b + c + d = 0. Prove that


(ab + ac + ad + bc + bd + cd)2 + 12 ≥ 6(abc + abd + acd + bcd) .

6. Let ABCDEF be a onvex hexagon su h that AD = BC + EF ,


AB CD EF
BE = AF + CD , and CF = DE + AB . Prove that = = .
DE AF BC

Next is the 50th Mathemati al Olympiad of the Republi of Moldova,


and problems of the 10th form. We again thank Robert Morewood, Canadian
Team Leader to the 2006 IMO in Slovenia, for olle ting them for us.
th
50 MATHEMATICAL OLYMPIAD OF THE
REPUBLIC OF MOLDOVA
th
10 Form

Mar h 2006

1 . Let a, b, and c be the side lengths of a right triangle with hypotenuse of


length c, and let h be the altitude from the right angle. Find the maximum
value of ca +
+b
h
.

2 . Let n ≥ 2 be an integer and M = {0, 1, 2, . . . , n − 1}. For a nonzero


integer a let fa : M → M where fa (x) is the remainder of ax upon division
by n. Give a ne essary and suÆ ient ondition for the fun tion fa to be
bije tive and prove that an(n−1) − 1 is divisible by n2 whenever n is a prime
number.
3 . The quadrilateral ABCD is ins ribed in a ir le. The tangents to the ir le
at A and C interse t at a point P not on BD and su h that P A2 = P B · P D.
Prove that BD passes through the midpoint of AC .
378

4. Find all values of the real number a for whi h the following equation has
a unique solution:
2x2 − 6ax + 4a2 − 2a − 2 + log2 2x2 + 2x − 6ax + 4a2


= log2 x2 + 2x − 3ax + 2a2 + a + 1 .




5. Let n be a positive integer and let x1 , x2 , . . . , xn be real numbers in the


interval 14 , 23 . Find the minimum value of the expression


     
1 1 1 1 1 1
log1.5x1
2

36x22
+ log1.5x2
2

36x23
+ · · · + log1.5xn
2

36x21
.

When is this value attained?


6 . Triangle ABC is isos eles with AC = BC and P is a point inside the
triangle su h that ∠P AB = ∠P BC . If M is the midpoint of AB , prove that
∠AP M + ∠BP C = 180◦ .

7 . The interior angles of a onvex o tagon are all equal and all side lengths
are rational numbers. Prove that the o tagon has a entre of symmetry.
8 . Let M = {x2 + x | x is a positive integer}. For ea h integer k ≥ 2 prove
that there exist a1 , a2 , . . . , ak , bk in M su h that a1 + a2 + · · · + ak = bk .

We also give the problems of the Se ond and Third Team Sele tion Tests
for the IMO 2006 from Chisinau, Moldova, Mar h 25-26, 2006. Thanks again
go to Robert Morewood, Canadian Team Leader to the 2006 IMO in Slovenia,
for olle ting them for us.
REPUBLIC OF MOLDOVA INTERNATIONAL
MATHEMATICAL OLYMPIAD
Se ond Team Sele tion Test

Mar h 25, 2006 (Chisin


 au)


1 . Let {an }∞
n=0 be a sequen e, with a0 = 2, a1 = 1, and an+1 = an + an−1
for n ≥ 1. Prove that a59 divides the number (a30 )59 − 1.
2. The point P lies in the interior of the ir le C1 , whi h in turn lies in the
interior of the ir le C2 , and the point Q lies in the exterior of the ir le C2 .
Ea h line ℓi (i ≥ 3) is di erent from the line P Q, passes through P , and uts
the ir le C1 at the points Ai and Bi ; and the ir um ir le of triangle QAi Bi
uts the ir le C2 at the points Mi and Ni . Prove that all the lines Mi Ni are
on urrent.
379

3. Let a, b, c be the side lengths of a triangle and let s be the semiperimeter.


Prove that
s s s
(s − b)(s − c) (s − c)(s − a) (s − a)(s − b)
a + b + c ≥ s.
bc ac ab

4. Let n be a positive integer and let A = {1, 2, . . . , n}. Determine the


number of distin t solutions of the equation
X ∪ Y ∪ Z = A,

where two solutions are onsidered the same if they di er only in the ordering
of X , Y , and Z .
Third Team Sele tion Test

Mar h 26, 2006 (Chisin


 au)


5 . The point P is in the interior of triangle ABC . The rays AP , BP , and CP


ut the ir um ir le of the triangle at the points A1 , B1 , and C1 , respe tively.
Prove that the sum of the areas of the triangles A1 BC , B1 AC , and C1 AB
does not ex eed s(R − r), where s, R, and r are the semiperimeter, the
ir umradius, and the inradius of triangle ABC , respe tively.
6 . Let n ≥ 2 be an integer and let X be a set with n + 1 elements. The
sequen es (a1 , a2 , . . . , an ) and (b1 , b2 , . . . , bn ) of distin t elements from the
set X are alled separated if ai = bj for some i 6= j . Determine the maxi-
mum number of ordered sequen es of n elements from X su h that any two
of them are separated.
7 . Let a, b, and c be positive real numbers su h that abc = 1. Prove that
a+3 b+3 c+3
+ + ≥ 3.
(a + 1)2 (b + 1)2 (c + 1)2

8 . Let f (n) be the number of permutations (a1 , a2 , . . . , an ) of the set


{1, 2, . . . , n} su h that a1 = 1 and |ai − ai+1 | ≤ 2 for 1 ≤ i ≤ n − 1. Prove
that f (2006) is divisible by 3.

Next we give the problems of the Final Round, 2nd Grade High S hool of
the Youth Mathemati al Olympiad of the Aso ia ion, Venezolana de Compe-
ten ias Matemati as,
 written May 27, 2006. Thanks go to Robert Morewood,
Canadian Team Leader to the IMO in Slovenia, for olle ting them.
380

YOUTH MATHEMATICAL OLYMPIAD


Aso ia ion
 Venezolana de Competen ias Matemati as

Final Round, 2
nd Grade High S hool (May 27, 2006)

1. A positive integer has 223 digits and the produ t of these digits is 3446 .
What is the sum of the digits?
2. Find all solutions of the equation m2 − 3m + 1 = n2 + n − 1, where m
and n are positive integers.
3 . De ne the sequen e a1 , a2 , a3 , . . . as follows: let a1 = a2 = 1003;
a3 = a2 − a1 = 0; a4 = a3 − a2 = −1003; and in general an+1 = an − an−1
for any n ≥ 2. Compute the sum of the rst 2006 terms of the sequen e.
4 . Joseph, Dario, and Henry prepared some labels. On ea h label they wrote
one of the numbers 2, 3, 4, 5, 6, 7, or 8. David joined them and stu k one
label on the forehead of ea h friend. Joseph, Dario, and Henry ould not see
the numbers on their own foreheads, they only saw the numbers of the other
two. David said, \You do not have distin t numbers on your foreheads, and
the produ t of the three numbers is a perfe t square." Ea h friend then tried
to dedu e what number he had on his forehead. Could anyone dis over it?
5 . Let ABC be an isos eles triangle with ∠B = ∠C = 72◦ . Find the value
of ABBC− BC
. Hint: Consider the bise tor CD of ∠ACB .

As a nal problem set from our 2006 IMO le we turn to the 42nd Mon-
golian Mathemati al Olympiad, 10th Grade, written at Ulaanbaatar in April
and May of 2006. Thanks go to Robert Morewood, Canadian Team Leader
at the 2006 IMO in Slovenia, for olle ting them for us.
Unfortunately, our version of problem 5 below was in omplete, and we
ould not remedy this in time. We will print this problem in a future issue
as soon as we lo ate it.
nd
42 MONGOLIAN MATHEMATICAL OLYMPIAD
th
10 Grade

April 28-May 4, 2006 (Ulaanbaatar)

1. Let a, b, c, d, e, and f be positive integers satisfying the relation


ab + ac + bc = de + df + ef , and let N = a + b + c + d + e + f .
Prove that if N | (abc + def ), then N is a omposite number.
2 . Let m and n be positive integers. In how many di erent ways an one
ll in an n × n grid with nonnegative integers su h that the sum of every n
numbers from di erent rows and olumns is m?
381

3. Cir les ω , ω0 have distin t entres and ω0 lies inside ω . The four ir les
ω1 , ω2 , ω3 , and ω4 ea h tou h ω internally and ω0 externally. Prove that the
interse tion points of the external tangents of ωi and ωj (i 6= j ) are ollinear.
4 . Let p and q be distin t prime numbers and let a and b be distin t positive
integers. Prove that there exists a prime number r su h that r | (apq − bpq )
and r ≡ 1 (mod pq).
5 . Omitted.
6 . Let a, b, c be positive real numbers su h that a + b + c = 1. Prove that
r r r
1−a 1−b 1−c
(1 + a) + (1 + b) + (1 + c)
a b c

3 3 (1 + a)(1 + b)(a + c)
≥ ·p .
4 (1 − a)(1 − b)(1 − c)

Now we give readers' solutions to problems in the September 2008


Corner and the X Bosnian Mathemati al Olympiad given at [2008 : 284{285℄.
1 . Let H be the ortho entre of an a ute-angled triangle ABC . Prove that
the midpoints of AB and CH and the interse tion point of the interior bi-
se tors of ∠CAH and ∠CBH are ollinear.
Solved by Miguel Amengual Covas, Cala Figuera, Mallor a, Spain;
George Apostolopoulos, Messolonghi, Gree e; Oliver Geupel, Bruhl,  NRW,
Germany; D.J. Smeenk, Zaltbommel, the Netherlands; and Titu Zvonaru,
Comane
 sti, Romania. We give the version of Geupel.
Let us denote α = ∠CAB ,
β = ∠ABC , γ = ∠BCA, and let C
A′ = AH ∩ BC , B ′ = BH ∩ AC . .....
............
Let M , N , O be the midpoints of .... ..... .......
. ....
segments AB , A′ B ′ , CH , respe -
.
... ... ....
β ..... ...... .....
.... A′
tively, let P = CH ∩ A B , and let O
....
′ ′ .
. ..... ...
α
.. ..
... ...N
.
... ..... ......................
..

Q be the interse tion point of the


........
.
. P ............ ...
... .
...
.. ...
. ...
.... .....
..
.. .. .. ....
bise tors of ∠CAA and ∠CBB , ..................... ..... ...........
.
′ ′
....
′ .. ......... .. .... ..
B .. .................. ...
..
. . ....
.....
respe tively. We are to show that . . ....
. ... ....
.... H
the points M , O, Q are ollinear.
... .....
.
. . ....
... ... ....
It is a well-known fa t that the
... .....
........α β
... ....
... ..
midpoints of the diagonals of a om-
................................................................................................................................
A M B
plete quadrilateral are ollinear.
Applying this to the omplete quadrilateral generated by the quadrilateral
A′ CB ′ H , we see that the points M , N , and O are ollinear. Due to the fa t
382

that ∠HA′ C = ∠CB ′ H = 90◦ , the quadrilateral A′ CB ′ H is y li with


ir um entre O.
Hen e, ON ⊥ A′ B ′ , whi h implies that
∠HOM = ∠P ON = 90◦ − ∠N P O = 90◦ − ∠A′ P C
90◦ − 180◦ − ∠P A′ C − ∠A′ CP

=
= ∠P A′ C + ∠BCH − 90◦
α + 90◦ − β − 90◦ = α − β ;

=

thus,
∠AM O = 90◦ − ∠HOM = 90◦ − α + β . (1)
Sin e ∠BAA′ = 90◦ − β and ∠A′ AC = 90◦ − γ , we have
1
∠BAQ = ∠BAA′ + ∠A′ AC
2
1 1
90◦ − β + 90◦ − γ = 135◦ − β − γ.

=
2 2
Similarly, ∠ABQ = 135◦ − α − 12 γ . Hen e,
∠AQB = 180◦ − ∠BAQ − ∠ABQ
1 1
   
= 180◦ − 135◦ − β − γ − 135◦ − α − γ
2 2
= α + β + γ − 90 = 90 .
◦ ◦

We dedu e that △AM Q is isos eles with AM = M Q. Thus,

1
 
◦ ◦ ◦
∠AM Q = 180 − 2∠M AQ = 180 − 2 135 − β − γ
2
= 90◦ − α + β . (2)
From (1) and (2) it is readily seen that ∠AM O = ∠AM Q, that is, the points
M , O , and Q are ollinear. This ompletes the proof.

2. Let a1 , a2 , and a3 be nonnegative real numbers with a1 + a2 + a3 = 1.


Prove that
√ √ √ 1
a1 a2 + a2 a3 + a3 a1 ≤ √ .
3

Solved by George Apostolopoulos, Messolonghi, Gree e; Mi hel Bataille,


Rouen, Fran e; George Tsapakidis, Agrinio, Gree e; and Titu Zvonaru, Comane
 sti,
Romania. We give the solution by Bataille.

Sin e the fun tion f (x) = x is on ave on [0, ∞) and a1 +a2 +a3 = 1,
Jensen's Inequality yields
√ √ √ √
a1 a2 + a2 a3 + a3 a1 ≤ a1 a2 + a2 a3 + a3 a1 . (1)
383

Now, from the well-known inequality a21 + a22 + a23 ≥ a1 a2 + a2 a3 + a3 a1 ,


we obtain
3(a1 a2 + a2 a3 + a3 a1 ) ≤ a21 + a22 + a23 + 2(a1 a2 + a2 a3 + a3 a1 )
= (a1 + a2 + a3 )2 = 1 ,

so that a1 a2 + a2 a3 + a3 a1 ≤ 13 . From (1), it follows that


√ √ √ 1
a1 a2 + a2 a3 + a3 a1 ≤ √ .
3

4 . Given are a ir le and its diameter P Q. Let t be a tangent to the ir le,


tou hing it at T , and let A be the interse tion of the lines t and P Q. Let p
and q be the tangents to the ir le at P and Q respe tively, and let
P T ∩ q = {N } and QT ∩ p = {M } .
Prove that the points A, M , and N are ollinear.
Solved by Miguel Amengual Covas, Cala Figuera, Mallor a, Spain;
Mi hel Bataille, Rouen, Fran e; D.J. Smeenk, Zaltbommel, the Netherlands;
and Titu Zvonaru, Comane
 sti, Romania. We give the write-up of Amengual
Covas.
Equivalently, we will prove that: Let T be a point of a ir le with a
given diameter P Q. Let p and q be the tangents to the ir le at P and Q,
respe tively, and let
P T ∩ q = {N } , QT ∩ p = {M } , and M N ∩ P Q = {A} .
Then AT tou hes the given ir le at T .
...
... p q ...
....
...
M
... ....
... ...
.
. ...
.
............................................................................... ...
......
......... ... ........... ...........
.... ....
......... ... .... ...... ........
.
.....
. .
.. .
.. .... ..... . .. ... ....... ....
... . .... ...... . ....... ...
.
...... ...
. .. . ...... .. ..... ....
... ..
. ...
.. .
... .... . ...... .
..... ....
..... . ..... ....... ..... ....
.
... .... ..... .. ..... .... ....
.... ... ..... ........ ..... ...
... ... .... .. ..... ....
..
. ... ... ....... ..
...
...
. ... ...
... .... .... ....... .. . ....
... ... ..... . .. ........ . ....
... ... ...
...
......
. .. ...
... ... .. ... .. ....
... ... .. ....... ..
... ...
... .. ........ . ....
.. .... . ... .. ...... .....
. . ..
U N
... ..... ...
. .... . . .
..... ...
.... ..................
t
... .. . . ... . .
... .......... ... . ....
..... .......... ... ... ... .....................................
.. ... .... .......... ........ ... ...... ..........
T
... ... ..........
.. .. ..........
.. .
...
.. .
................. ............ .... ......... ..........
........... . . ..... ...
..
..
... .......... ...... ........................ .
..... ..... ...
.. .... ...
..........................................................................
. ... ..... ...
..........
.. ... ........... ..... ..
....
......... .....
.
... . ........ ............................................ .......
.....
V
... ...... ...... .
. ......
... ... . ..... .
.....
....... .......... ..... ........................ .... ..........
... .. . ..
.... ....... ... ... ..... .............. . ...
... .... ..
. .. ..
.. .... .... .. ........... . .
. ... .........
.... ... ..... .......... . ....... ... . . . . .
..
. ... .....
.... . ...
.
. . .......... ..... ... .... ... .... ...
.. . .
..........
..
.... .. . .....
..... . .... .. ... .... ... .. ... .......... .....
..... .... .... ...... .. .. .... .. . . . . . .
. ..................
. .....
..... .
. .. .......... ...... ... ... .. . .
. .. .......... .........
.......
........ ......... .......... .... ....... .
.. ....
.
... ....
.. . .
. ..... .......... .....
......... ..... ........... ... ...... . . ..... ... ..... ..... .......... ....
........... .. ..
. . ......... .... .. . ... .......... .....
................. .. ....... ........... .. . .. ........ .... .
...................................................................................................................................................................................................................................................................................................................................................
Q
..
P ..
...
.. O .
..
.
....
A
.. ..
... ..
... ..
... ....
... ..
.... ...
.... ....
..... ....
.......
........ .
....
......
........... ..... .
.....................................................
384

Let AT ∩ p = {U }. Then, by Ceva's Theorem,


M U P Q AN
· · = 1.
U P QA N M
PQ NM
But p ⊥ P Q and q ⊥ P Q, so that pkq, implying that QA
=
AN
. Hen e,
MU
UP
=1 and M U = UP .
Let AT ∩ q = {V }. Sin e
NV AV VQ
= = ,
MU AU UP
we obtain N V = V Q.
Thus, U and V are the midpoints of M P and N Q, respe tively.
Sin e ∠P T Q is a right angle, so also ∠M T P = 90◦ = ∠N T Q. Hen e
both ir les onstru ted with M P and N Q as diameters pass through T .
Sin e the distan e between their entres U and V equals the sum of
their radii, these ir les are externally tangent to ea h other at T .
Let their ommon tangent at T interse t P Q at O. This tangent is
perpendi ular to the line U V through the entres, and sin e
P O = OT = OQ ,

the point O is the midpoint of P Q, making OT a radius of the given ir le in


our problem.
Thus, OT ⊥ AT , and the on lusion follows.
5 . A permutation of the set {1, 2, . . . , n} satis es the
(a1 , a2 , . . . , an )
2
inequality aak ≤ k+2 for ea h k = 1, 2, . . . , n − 1. Prove it is the
k+1
identity.
Solution by Titu Zvonaru, Comane
 sti, Romania.
Suppose that an 6= n. Then an ≤ n − 1, and we have
a2n−1 ≤ (n + 1)an ≤ (n + 1)(n − 1) = n2 − 1 ,

hen e, an−1 ≤ n − 1.
Now, sin e a2n−2 ≤ nan−1 ≤ n2 − n, we dedu e that an−2 ≤ n − 1.
In general, if 1 ≤ k ≤ n − 1 and ak+1 ≤ n − 1, then
a2k ≤ (k + 2)ak+1 ≤ (n + 1)(n − 1) = n2 − 1 ,

and we dedu e that ak ≤ n − 1.


From an 6= n it then follows that ak < n for ea h index k. However,
n ∈ {a1 , a2 , . . . , an }, a ontradi tion!
Therefore, an = n, and similarly, an−1 = n − 1, . . . , a2 = 2, a1 = 1,
whi h means that (a1 , a2 , . . . , an ) is the identity.
385

6 . Let a, b, and c be integers su h that


a b c
+ + = 3.
b c a
Prove that abc is a perfe t ube.
Solved by George Apostolopoulos, Messolonghi, Gree e; Mi hel Bataille,
Rouen, Fran e; George Tsapakidis, Agrinio, Gree e; and Titu Zvonaru, Com-
ane
 sti, Romania. We give the generalization of Bataille.
a b c
We generalize and show that if b
+ +
c a
= n for some integer n,
then abc is a perfe t ube. Assume that ab + bc + ac = n.Certainly abc is
a perfe t ube if |abc| = 1. From now on, we suppose that |abc| > 1. It
suÆ es to prove that for any prime p dividing abc, the exponent of p in the
prime fa torization of abc is a multiple of 3.
Let p be su h a prime. There exist nonnegative integers r, s, and t with
(r, s, t) 6= (0, 0, 0) and su h that a = pr a1 , b = ps b1 , and c = pt c1 , where
a1 , b1 , and c1 are integers not divisible by p. We have to show that r + s + t
is divisible by 3. From the hypothesis, we obtain
pr−s a21 c1 + ps−t b21 a1 + pt−r c21 b1 = na1 b1 c1 . (1)
Let N be the number of negative integers among r − s, s − t, t − r, and
onsider the possibilities for N .
First, note that (r − s) + (s − t) + (t − r) = 0, so N = 3 annot
hold. Also, N = 1 annot hold either: if we had, say, r − s, s − t ≥ 0 and
t − r < 0, then the relation (1) would imply

c21 b1 = pr−t na1 b1 c1 − pr−s a21 c1 − ps−t b21 a1 ,




ontradi ting the fa t that p does not divide c21 b1 .


If N = 0, then r − s, s − t, and t − r are nonnegative and sum to zero,
hen e r = s = t and r + s + t is a multiple of 3.
If N = 2, then suppose (for example) that s−t, t−r < 0 and r−s > 0.
Then (1) yields
b21 a1 c21 b1
pr−s a21 c1 + + = na1 b1 c1 .
pt−s pr−t

If, in addition, we had t − s 6= r − t, then by multiplying by pmax(t−s,r−t)


we would dedu e that p divides b21 a1 (if t − s > r − t) or p divides c21 b1
(if r−t > t−s), thus arriving at a ontradi tion in either ase. Consequently,
t − s = r − t and r + s + t = 3t, a multiple of 3.
This last ase ompletes the analysis.
386

We next look at solutions from our readers to the problems of the


I elandi Mathemati al Contest 2004{2005 (Final Round) given in the Corner
at [2008 : 285℄.
1 . How many subsets with three elements an be formed from the set
{1, 2, . . . , 20}
so that 4 is a fa tor of the produ t of the three numbers
in the subset?
Solved by George Apostolopoulos, Messolonghi, Gree e; and John Grant
M Loughlin, University of New Brunswi k, Frederi ton, NB. We give the
solution of M Loughlin.
There are 20 = 1140 subsets with 3 elements.

3
All of these subsets have elements that when multiplied will have 4 as
a fa tor, ex ept in two ases:
• all elements are odd, of whi h there are 10
3
= 120 subsets;
• two elements are odd and the
 5third element is even but not a multiple
of 4, of whi h there are 10
2 1
= 225 subsets.

Therefore, 1140 − (120 + 225) = 795 subsets meet the requirements.


2. Triangle ABC is equilateral, D is a point inside the triangle su h that
DA = DB , and E is a point that satis es the onditions ∠DBE = ∠DBC
and BE = AB . How large is the angle ∠DEB ?
Solved by Miguel Amengual Covas, Cala Figuera, Mallor a, Spain; and Titu
Zvonaru, Comane
 sti, Romania. We give Zvonaru's answer.
Sin e DA = DB , the point D lies A
on the perpendi ular bise tor of AB , ........

hen e ∠DCB = 30◦ . We have ...


.
... .......
... .. ...
... .... .....
.. ..
... ...
..
... .. ...

BC ;
... ..
...
BE =
.. ....
E
. .
.... ...
..
...
.. ... ...
.

∠DBC ;
.. ...
.. .. ... ....
... .................
∠DBE = ... ...
. . .. ...... .
.. ... .................. ....
.
......... .....

...............
... .......

BD .
.....

. .. ...... .. . .
.....

........ ....
... ........
BD =
........ ...
.. ........ ....... .. ... ...
... ........ ............. .... ... .....
. . ... ..............
. .
. .. .... ...
..... .
........ .
... ....... ........ .... ... ...
...
It follows that △BDE and △BDC
........ ........ ..... ...
....
. .............
. . ..
..
............
. ...
...
...
... ........ ... ..... ......... ........
. ...
D
. ... ........ . .
... ..
.

are ongruent (side-angle-side), hen e


... .... . ...... ..
.. ......... ........
..
.........
...
. . ............. ............... ..
. . ....... ..
.......... ....
................................................. ........ ...
....................... .. ... .
.......................................................................................................................................................................................
...

B C
∠DEB = DCB = 30◦ .

3 . Find a three-digit number n that is equal to the sum of all the two-digit
numbers that an be formed by using only the digits of the number n. (Note
that if a is one of the digits of the number n, then aa is one of the two-digit
numbers that an be formed.)
387

Solution by Titu Zvonaru, Comane sti, Romania.


Let a, b, and c denote distin t digits. The following ases are ruled out:
(a) n = aaa = aa ⇒ 111a = 11a ⇒ 100a = 0; no solution in this ase.
(b) n = aab = aa + bb + ab + ba ⇒ 110a + b = 22a + 22b ⇒ 88a = 21b.
There is no solution here, sin e 11 divides 88a but does not divide 21b.
( ) n = aba = aa + bb + ab + ba ⇒ 79a = 12b; no solution as b is not
divisible by 79.
(d) n = baa = aa + bb + ab + ba ⇒ 78b = 11a; no solution as 78b is not
divisible by 11.
Thus, n = abc = aa + bb + cc + ab + ba + bc + cb + ac + ca and we
have abc = 33(a + b + c).
It follows that 3 divides abc, hen e 3 divides a+b+c and then 9 divides
33(a + b + c). Now 9 divides abc, hen e 9 divides a + b + c and n an only
be one of
33 · 9 = 297 ; 33 · 18 = 594 ; 33 · 27 = 891 .

Therefore, 594 = 55 + 99 + 44 + 59 + 95 + 94 + 49 + 54 + 45 is the


only solution.
4. Transportation between six ities is su h that between any two ities
there is either a bus or a train but not both of these. Show that among these
six ities there are three ities that are linked either only by buses or only by
trains.
Solved by George Apostolopoulos, Messolonghi, Gree e; and John Grant
M Loughlin, University of New Brunswi k, Frederi ton, NB. We give the
response of M Loughlin.
Consider six ities A, B , C , D, E , and F . There must be either at
least three train links to ity A or at least three bus links to ity A. We will
assume three train links without loss of generality. Let us suppose that A is
onne ted to ea h of B , C , and F by train, as indi ated in the diagram.
Assume that no three ities an
be linked by train or bus. Then CR, A ................................T B
BC , and BF annot be train links, as
..................................... .
.......
... ............. ...... ....

that would result in linking three ities


. ......... ...... ..
.. ........ .
....
.
.. ..
... ........
......
...... ..
..

by train. .... ........


........ T ...
. ..
T...
.
.... ..
. ........ .....
. ..
.

Using dashed segments for bus


. .
. ..
. ........
.
. ..... ..
.
. ..
. ... ........ ..
..
. ..... ........
links, it now happens that B , C , and F
. . . ..
.
. ..
. .... ........ ..
.. .....
. ... ........ ..
.
.. ......
...................................................................................................................

are all onne ted by a bus link. Hen e, F C


using proof by ontradi tion, it has
been shown that there are three ities linked only by buses or only by trains.
388

Comment. This is a tually the game of SIM in disguise. The game requires
players to alternately (using one olour ea h) join two verti es with a seg-
ment. The player who rst forms a triangle with all segments in their olour
loses. The game annot end in a tie as shown in this problem.
1
5 . Determine whether the fra tion 2005 an be written as a sum of 2005
di erent unit fra tions. (A unit fra tion is a fra tion of the form n1 , where n
is a natural number.)
Solution by John Grant M Loughlin, University of New Brunswi k, Freder-
i ton, NB.
1
Yes, 2005
an be written as the sum of 2005 distin t unit fra tions.
1 1 1
For all positive integers m, m =
m+1
+
m(m + 1)
. If m ≥ 2, this
an be employed to in rease the number of unit fra tions in a sum without
limit:
1 1 1
= +
2005 2006 (2005)(2006)
1 1 1
= + +  
2006 (2005)(2006) + 1 (2005)(2006) (2005)(2006) + 1

and so on!
6 . Let h be the altitude from A in an a ute triangle ABC . Prove that
(b + c)2 ≥ a2 + 4h2 ,

where a, b, and c are the lengths of the sides opposite A, B , and C respe -
tively.
Solved by Miguel Amengual Covas, Cala Figuera, Mallor a, Spain; George
Apostolopoulos, Messolonghi, Gree e; Mi hel Bataille, Rouen, Fran e; Oliver
Geupel, Bruhl,
 NRW, Germany; and Titu Zvonaru, Comane  sti, Romania. We
give the solution of Amengual-Covas.
The triangle ABC need not be a ute, as the proof will show.
Let wa be the length of the internal bise tor of angle A.
From the known relations
2bc A
wa = cos and 4bc ≤ (b + c)2 ,
b+c 2

we obtain
4bc A A
wa2 = bc · cos2 ≤ bc · cos2 .
(b + c)2 2 2
389

However, h ≤ wa , so we have
A 1 + cos a
h2 ≤ wa2 ≤ bc · cos2 = bc ·
2 2
 
b2 + c2 − a 2
1+
2bc (b + c)2 − a2
= bc · =
2 4
and (b + c) ≥ a + 4h follows, as desired.
2 2 2

Equality holds only if b = c.

To omplete this number of the Corner we begin to look at solutions


from our readers to problems given in the O tober 2008 issue, starting with
the Olimpiada Matemati a
 Espanola
~ 2005 given at [2008 : 341{342℄.
3. Let r, s, u, and v be real numbers. Prove that
1
min{r − s2 , s − u2 , u − v 2 , v − r 2 } ≤
4
.
Solved by George Apostolopoulos, Messolonghi, Gree e; Mi hel Bataille,
Rouen, Fran e; and Titu Zvonaru, Comane  sti, Romania. We give Bataille's
write-up.
Suppose that min r − s2 , s − u2 , u − v2 , v − r2 > 14 . Then ea h of


the numbers r − s2 , s − u2 , u − v2 , and v − r2 is greater than 14 , hen e


1
(r + u) − s2 + v 2 (1)

>
2
and 1
(s + v) − r 2 + u2 . (2)

>
2
Sin e (s + v)2 ≤ 2 s2 + v 2 , inequality (1) yields


2(r + u) − (s + v)2 > 1


and similarly from inequality (2) we obtain
2(s + v) − (r + u)2 > 1 .
Let a = r + u. Then,  2
1 + a2
2a > 1 +
2
,
whi h is equivalent to the inequality a4 + 2a2 − 8a
 + 5 < 0, a ontradi tion,
sin e a4 + 2a2 − 8a + 5 = (a − 1)2 a2 + 2a + 5 is nonnegative for all real
numbers a. The result follows.

That ompletes the Corner for this issue. Send me your ni e solutions
and generalizations!
390

BOOK REVIEWS
Amar Sodhi

Sink or Float? Thought Problems in Math and Physi s


By Keith Kendig, Mathemati al Asso iation of Ameri a, 2008
ISBN 978-0-88385-339-9, hard over, 375+xiii pages, US$59.95
Reviewed by Nora Franzova, Langara College, Van ouver, BC
May the title not onfuse you! Only a small part of the book is geared
towards Ar himedes enthusiasts. A tually there are ten di erent ategories
of problems spanning geometry, numbers, probability, lassi al me hani s,
astronomy, linear algebra, ele tri ity and magnetism, the leaking tank, heat
and wave phenomena, and Ar himedes' Prin iple. These ategories are neatly
separated into groups, where transitioning between math and physi s prob-
lems happens without any a tual transition. Like walking on a Mobius strip,
there is only one side to it: the hallenge and joy of solving problems.
The book opens with \A Sampler" of problems that immediately pull
you in and, as their diÆ ulty in reases, you realize that this is a treasure to
be savoured slowly, sin e ea h problem, no matter how short and sweet it
seems, an lead to surprising truths.
Ea h of the ten se tions of the book starts with an introdu tion that
brings up interesting histori al information and lays down notation, basi
de nitions, and o asional theorems that are needed in the se tion. Short
examples are in luded. These are not aimed to tea h the material, just to
bring the reader to the starting point from whi h the author is observing
that parti ular subje t area.
Problems are not numbered, but ea h has a title, a name that is already
a part of the story. Ea h title is followed by a short des ription, and then
a list of possible answers is given in a multiple hoi e format with most of
the answers being oh-so-tempting. In between individual problems, addi-
tional histori al and theoreti al information is provided in learly separated
darkened frames, whi h seem very inviting. Kendig revisits several questions
more than on e, so we en ounter not only a problem alled \Friday the 13th ",
but also \Friday the 13th Again" and \Friday the 13th One More Time". None
involves any bla k ats.
The author states: \I hope that this book will serve as a springboard {
either individually or in lassroom { for mathemati al thought, ase testing,
dis ussions, arguments." The rst step an be taken by following his advi e
from the prefa e. The author re ommends one read the problem, think about
it, then lose the book and think about it some more. Only then should one
turn the pages to the designated pla e where the omplete solution an be
found. The solutions are more than omplete; many also in lude short side
dis ussions about related questions. Very helpful are the graphi al images
that are part of many of the problems and their solutions. These images talk
to the reader through questions in framed \bubbles".
391

The book brought forth an immediate urry of interest and omments


like \every problem is a gem" or \this is so neat," from family and guests
after it had be ome a staple on our o ee table. One might say that a \math
book" is not o ee table material, but the problems in the book are attra -
tively presented, and fairly simple to understand, so a lay person (let's say
a \mathemati al muggle⋆ ") an be tempted to approa h a problem like the
following: \Is it possible to stand at just the right pla e on the earth so that
the moon's res ent looks like a smiley fa e instead of being more verti al?"
And a mathemati al wizard (just to sti k with Harry Potter terminology) will
humbly turn the pages, pretending only mild interest while still trying to
solve that probability problem from the beginning. A math wizard knows
that these questions only look inno ent. In reality they hide a lot of deep
truths, ready to be un overed again and again.
Even though the book is from the Dol iani Mathemati al Expositions
series, it de nitely makes an ex ellent o ee table book. Many of the prob-
lems an lead to lively dis ussions and arguments for any group of urious
individuals. For a similar reason the book would make an ex ellent prize to
be given to winners of math ontests or s ien e fairs. And, sin e it is not to be
read qui kly, professors, graduate and undergraduate students, or even high
s hool students and their tea hers, will be oming ba k to it to rejuvenate
their uriosity and wonder about nature's mysteries.
We all know that solving a good problem brings us not only satisfa tion
but also the joy of understanding. And that an only be repeated by nding
and solving another interesting problem and then another ... .
At this point, all that is left to the thoughtful person is to open Keith
Kendig's book and start solving problems.

Muggle is the word used in the Harry Potter series of books by J.K. Rowling
to refer to a person who la ks any sort of magi al ability.

Number Theory Through Inquiry


By David C. Marshall, Edward Odell, and Mi hael Starbird, Mathemati al
Asso iation of Ameri a, Washington, DC, 2007
ISBN 978-0-88385-751-9, hard over, 140+x pages, US$51.00
Reviewed by Jeff Hooper, A adia University, Wolfville, NS
Although we an learn mathemati s by reading texts or by sitting in
lasses, mathemati s is not a spe tator sport: most of us learn new math-
emati al ideas by a tively working through examples and problems. One
re ent, and very laudable, trend in the tea hing of mathemati s has been to-
wards the adoption of tea hing methods whi h push students towards
be oming more independent in their mathemati al thinking. An important
example of this is the method known as Inquiry-Based Learning (or IBL for
short).
The basi notion behind IBL-related methods is to have the students'
ideas and questions drive the development of the material, with the tea her
392

a ting as fa ilitator. In other words, in its purest form, students would work
through examples, use them to suggest onne tions among ideas, formulate
statements of onje tures, and then prove these results, with the instru tor
there only to help push things along when needed. In prin iple, be ause
students must work out mu h of the details and proofs themselves, they
gain a deeper understanding of the material, and more importantly develop
a mu h higher level of mathemati al skill. Of ourse, this pure form of IBL,
as I've des ribed it, would be extremely taxing for the students involved,
and far too time- onsuming, so in pra ti e the approa h gets modi ed so
that students are at least provided, as the ourse moves along, with pie es
of an outline: examples to onsider and statements of results whi h must be
proved. This skeletal outline ensures that the inquiry moves in appropriate
dire tions.
In mathemati s, a version of this approa h was hampioned in the 1950s
and 60s by R.L. Moore of the University of Texas at Austin. Moore used this
method to tea h sele ted lasses of top students, many of whom went on
to earn Ph.D. degrees and be ome well-established resear hers. Moore's
method was very pure, and students often had to on o t their own de ni-
tions of ne essary on epts.
The authors have been involved in a `modi ed-Moore' approa h to
tea hing mathemati s, whi h is aimed at a wider audien e, and this book is
an outgrowth of their ourses. The text has two main goals in mind: tea h-
ing the fundamental material overed in a standard undergraduate number
theory ourse, and tea hing students to be ome independent mathemati al
thinkers.
The book overs a fairly standard sele tion of topi s: divisibility and
greatest ommon divisors, primes and their distribution, the arithmeti of
ongruen es, Fermat's Little Theorem and Euler's generalization, appli a-
tions to publi -key ryptography and RSA, higher-order ongruen es and
primitive roots, quadrati re ipro ity, Pythagorean triples, sums of squares,
Pell's equation and introdu tory Diophantine approximation, and primality
testing. So the outline is mu h like what one would nd in many popular
elementary number theory texts.
What makes this text more unusual is the style. The authors use this
material for introdu ing students to proofs and abstra t mathemati s, so the
rst ouple of hapters are more leisurely than the rest, and there are many
omments to help the reader along. More importantly, be ause the entire
point of this book is its use in an IBL-style number theory ourse, the nar-
rative in ludes de nitions, exer ises for students to work through, as well
as statements of lemmas, propositions, theorems, and orollaries. But no
proofs or solutions are in luded. The intent is that the reader will work
out the proofs of these ideas for herself. The authors also in lude numerous
questions for students to ponder, many of them open-ended. These generally
require students to think more deeply about the material just en ountered,
and often ask students to formulate onje tures and prove them. Many se -
tions on lude with a `Blank-Paper Exer ise'. These ask students to return,
393

after a day or two away from the material, and try to outline the ideas in as
mu h detail as possible without looking at the text.
Other number theory texts have been written from a problems point-of-
view. Three whi h an be found in many libraries are: R.P. Burn's A Pathway
into Number Theory, Joe Roberts' Elementary Number Theory: A Problem
Oriented Approa h, and Sierpinski's
 250 Problems in Elementary Number
Theory.
So who would bene t from this text? On the ba k over the book sug-
gests its use as a text for a proof-transition or introdu tory abstra t mathe-
mati s ourse, or for independent study. But I must add a aveat to this last
suggestion: be ause there are no proofs or solutions, I'm not so sure of its
appropriateness for independent work. It would ertainly be useful in this
regard, but likely would require that the student or reader have some a ess
to help when they get stu k. I'm not omplaining here; it's lear that provid-
ing proofs and solutions would detra t onsiderably from its other purpose.
However, a good high-s hool student ould work through a text like this only
if she an depend on some a ess to tea her support.
As a professor, I found myself disappointed in one respe t. I've been
using a related approa h in some of my senior-level undergraduate ourses
with some su ess, and I was really looking forward to reading about the
authors' approa h. How do they use this material in their ourses? What
advi e would they have for an instru tor hoping to use this material? Most
texts treat these sorts of questions in the prefa e. Unfortunately, for some
reason the authors in lude no prefa e, and their introdu tory hapter, while
des ribing the text and speaking to its goals, doesn't address these questions
or explain how the material ould be used. Reading through the text fairly
arefully, the `Questions' and `Blank-Paper Exer ises' give some hint at the
approa h the authors use, but I would have wished for more here.
My small disappointment aside, this is a good text, and should er-
tainly be onsidered by anyone looking to experiment with an inquiry-based
approa h to the tea hing and learning of mathemati s.

Geometri Origami
By Robert Gerets hlager, Arbelos, 2008
ISBN 978-0-9555477-1-3, soft over, 198+xiii pages, US$24.99
Reviewed by Georg Gunther, Sir Wilfred Grenfell College, Corner Brook,
NL
Origami is the traditional Japanese art of paper folding whi h has been
pra tised in that ountry for over 400 years. Not surprisingly, this art form
with its stri t dependen e on a sequen e of pre ise folds, has long been of in-
terest to mathemati ians. In 1989, the rst International Meeting of Origami
S ien e and Te hnology was held; sin e that time there have been numerous
onferen es devoted to this burgeoning eld, and today the resear h litera-
ture is extensive and ever growing.
394

The book Geometri Origami provides a ni e balan e between theory


and pra ti al appli ation. If your interest is to fold a regular triskaide agon
(otherwise known as a 13-gon), you will nd detailed instru tions, a om-
panied by lear drawings, to guide you on your way. On the other hand, for
those whose interests are theoreti al, they will nd a lear exposition of the
mathemati s underlying this onstru tion.
This is a book whi h yields wonderful nuggets, regardless whether it
is read arefully from beginning to end, or whether it is skimmed super -
ially. Part I, whi h develops the underlying mathemati al theory, provides
a lear and areful dis ussion of the axioms of origami onstru tions, and
shows that these are more powerful than the orresponding axioms govern-
ing Eu lidean onstru tions. Spe i ally, origami onstru tions are powerful
enough to provide geometri solutions to the problem of solving the general
ubi equation (and, indeed, the general quarti as well), something that is
beyond the s ope of Eu lidean onstru tions. Hen e, as the author shows,
origami onstru tions enable one to solve two of the lassi al onstru tion
problems: dupli ating the ube and trise ting a general angle.
In Part II, the author gives lear step-by-step origami onstru tions
for a fas inating lass of problems. All of these results are bolstered by a
thorough dis ussion of the underlying theory. Mu h of this se tion fo uses
on origami onstru tions of regular polygons, and Gerets hlager ta kles this
from several dire tions. First, he explores the general question of whi h
n-gons are onstru tible (for example, he shows that if n = 5 · 2k · 3ℓ , then
the orresponding n-gon an be folded). Subsequently, the author devotes
one hapter to questions around the theory and onstru tion of the regular
pentagon, followed by a hapter exploring similar questions for the regular
heptagon. The book on ludes with a nal hapter des ribing how to fold the
regular nonagon, triskaide agon, 17-gon, and 19-gon.
This book is a delightful addition to a wonderful orner of mathemati s
where art and geometry meet in su h a fruitful union. Anyone with a working
knowledge of elementary geometry, algebra, and the geometry of omplex
numbers will have little trouble following the theoreti al dis ussions. It will
be of value to a wide range of audien es; new omers to the eld of mathemat-
i al paper-folding will nd, easily a essible, all the tools they need, while
experts will enjoy a well-written, beautifully illustrated referen e work.
395

PROBLEMS
Toutes solutions aux problemes dans e numero doivent nous parvenir au plus
tard le1er avril 2010 . Une e toile (⋆) apres le numero indique que le probleme a e te
soumis sans solution.
Chaque probleme sera publie dans les deux langues oÆ ielles du Canada
(anglais et fran ais). Dans les numeros 1, 3, 5 et 7, l'anglais pre edera le fran ais,
et dans les numeros 2, 4, 6 et 8, le fran ais pre edera l'anglais. Dans la se tion des
solutions, le probleme sera publie dans la langue de la prin ipale solution presentee.
La reda tion souhaite remer ier Jean-Mar Terrier, de l'Universite de
Montreal, d'avoir traduit les problemes.

3439 . Rempla ement. Propose par D.J. Smeenk, Zaltbommel, Pays-Bas.


Soit Γ un er le de entre O et de rayon R. La droite t est tangente a Γ
au point A, et P est un point sur t distin t de A. La droite ℓ, distin te de t,
passe par P et oupe Γ aux points B et C . Le point K est sur la droite AC
et tel que P KkAB , et le point L est sur la droite AB et tel que P LkAC .
Montrer que KL ⊥ OP .
3463 . Propose par Mi hel Bataille, Rouen, Fran e.
Soit Γ un er le de entre O et de rayon r, et soit P un point tel que
OP > r . Soit L l'ensemble de toutes les droites ℓ telles que P 6∈ ℓ et ℓ oupe
Γ aux points A, B de sorte que P A · P B = OP 2 − r 2 . Montrer que L est
un fais eau de droites on ourantes.
3464 . Propose par Mi hel Bataille, Rouen, Fran e.
Soit ABC un triangle ave ∠A = 90◦ et H le pied de la hauteur
abaissee
 de A. Soit J le point sur l'hypotenuse
 BC tel que CJ = HB et
soit K , L les proje tions respe tives de J sur AB , AC . Montrer que
2
 
M − ; AK, AL ≤ M (−2; AB, AC) ,
3
1/α
xα + y α
ou M (α; x, y) = .

2

3465 . Propose par Xavier Ros (etudiant)


 and Jose Luis Daz-Barrero, Uni-
versite Polyte hnique de Catalogne, Bar elone, Espagne.
∞ X

1 π2 1 3
Montrer que log 2.
X
< + +
i=1 j=1
ij(i + j) 6 2 4
396

3466 . Propose par Tuan Le, etudiant,


 Fairmont High S hool, Anaheim,

CA, E-U.

Soit x, y et z des nombres reels
 positifs tels que xyz ≥ 10 + 6 3.
Montrer que
y z x 1
3 2
+ 3 2
+ 3 2
≤ .
x+y +z y+z +x z+x +y 2

3467 . Propose par Tuan Le, etudiant,


 Fairmont High S hool, Anaheim,

CA, E-U.
Soit x, y et z des nombres reels
 positifs. Montrer que

r r
x3 + y 3 + z 3 xy + yz + xz
3 + 1.
3 3
+ 2 2 2

xyz x +y +z

3468 . Propose par Joseph DeVin entis, Salem, MA, E-U  ; Bernardo
Re aman,
 Institut Alberto Merani, Bogota, Colombie ; Peter Saltzman,
 ; et Stan Wagon, Ma alester College, St. Paul, MN, E-U.
Berkeley, CA, E-U 
Trouver tous les entiers positifs n pour lesquels on peut faire orres-
pondre a haque nombre pair de {2, 4, . . . , 2n} exa tement un nombre
impair de {1, 3, . . . , 2n − 1} de telle sorte que les sommes des n paires,
prises deux a deux, soient des nombres relativement premiers. Par exemple,
si n = 3, on a une solution, 2 + 3 = 5, 4 + 5 = 9, 6 + 1 = 7.
3469 . Propose par Mihaela Blanariu, College
 Columbia Chi ago, Chi ago,

IL, E-U.
Soit p ≥ 2 un nombre reel.
 Trouver la limite
√ √ √
1+ 2(2!)p + 3
3(3!)p + · · · + n n(n!)p
lim .
n→∞ (n!)p

3470 . Propose par Mihaela Blanariu, College


 Columbia Chi ago, Chi ago,

IL, E-U.
Soit p ≥ 2 un nombre reel.
 Trouver la limite
√ p √
3
p √ p
1+ 2! + 3! + · · · +
n
n!
lim .
n→∞ np+1

3471 . Propose par Catalin Barbu, Ba au, Roumanie.


Soit ABC un triangle a utangle et M , N , P les points milieu respe tifs
des petits ar s BC , CA, AB . Si [XY Z] designe  l'aire du triangle XY Z ,
montrer que [M BC] + [N CA] + [P AB] ≥ s(3r − R), ou s, r et R sont
respe tivement le demi-perim  etre,
 le rayon du er le ins rit et le rayon du
er le ir ons rit du triangle ABC .
397

3472 . Propose par Xavier Ros, etudiant,


 Universite Polyte hnique de
Catalogne, Bar elone, Espagne.
Soit x, y et z des nombres reels
 positifs tels que x ≤ y ≤ z . Montrer
que
1+y 1+z 1+x
     
log x · log + log y · log + log z · log ≥ 0.
1+z 1+x 1+y

3473 . Propose par Walther Janous, Ursulinengymnasium, Innsbru k,


Autri he, a la memoire
 de Jim Totten.
Soit c un nombre reel
 donne et tel que 0 < c ≤ 1.
(a) Montrer que pour tout nombre reel  positif µ, on a
2 µ 3
√ + p ≤ √ .
cµ + 1 c+µ 2 c +1

(b) ⋆. Trouver tous les nombres reels


 positifs λ tels que la relation
λ µ λ+1
√ + p ≤ √
cµ + 1 c+µ 2 c+1

soit veri
 ee  pour tous les nombres reels
 positifs µ.
3474 . Propose par Silouanos Brazitikos et Christos Patilas, Trikala, Gre e. 
Soit x, y et z des nombres q reels positifs tels que xyz = 1. Montrer si
oui ou non, on a l'inegalit
 e 2 3 xy + yz + z x + x + y + z ≥ 9.


[Ed. : Les proposeurs font remarquer que l'inegalit  e est appuyee


 par
des al uls sur ordinateur.℄
.................................................................
3439. Repla ement. Proposed by D.J. Smeenk, Zaltbommel, the Nether-
lands.
Let Γ be a ir le with entre O and radius R. Line t is tangent to Γ at
the point A, and P is a point on t distin t from A. The line ℓ distin t from t
passes through P and interse ts Γ at the points B and C . The point K is on
the line AC and su h that P KkAB , and the point L is on the line AB and
su h that P LkAC . Prove that KL ⊥ OP .
3463 . Proposed by Mi hel Bataille, Rouen, Fran e.
Let Γ be a ir le with entre O and radius r, and let P be a point with
OP > r . Let L be the set of all lines ℓ su h that P 6∈ ℓ and ℓ interse ts Γ
at points A, B su h that P A · P B = OP 2 − r2 . Show that L is a pen il of
on urrent lines.
398

3464 . Proposed by Mi hel Bataille, Rouen, Fran e.


Let ABC be a triangle with ∠A = 90◦ and H be the foot of the altitude
from A. let J be the point on the hypotenuse BC su h that CJ = HB and
let K , L be the proje tions of J onto AB , AC , respe tively. Prove that
2
 
M − ; AK, AL ≤ M (−2; AB, AC) ,
3
x + y α 1/α
 α
where .

M (α; x, y) =
2

3465 . Proposed by Xavier Ros (student) and Jose Luis Daz-Barrero, Uni-
versitat Polite ni a
 de Catalunya, Bar elona, Spain.
∞ X

1 π2 1 3
Prove that log 2.
X
< + +
i=1 j=1
ij(i + j) 6 2 4

3466 . Proposed by Tuan Le, student, Fairmont High S hool, Anaheim,


CA, USA.

Let x, y, and z be positive real numbers su h that xyz ≥ 10 + 6 3.
Prove that
y z x 1
+ + ≤ .
x+ y3 + z2 y+ z3 + x2 z+ x3 + y2 2

3467 . Proposed by Tuan Le, student, Fairmont High S hool, Anaheim,


CA, USA.
Let x, y, and z be positive real numbers. Prove that
s s
x3 + y 3 + z 3 xy + yz + xz √
3 + 1.
3 3
+ 2 2 2

xyz x +y +z

3468 . Proposed by Joseph DeVin entis, Salem, MA, USA; Bernardo


Re aman,
 Instituto Alberto Merani, Bogota,  Colombia; Peter Saltzman,
Berkeley, CA, USA; and Stan Wagon, Ma alester College, St. Paul, MN, USA.
Find all positive integers n for whi h one an mat h ea h even number
from {2, 4, . . . , 2n} with exa tly one odd number from {1, 3, . . . , 2n − 1}
so that the sums of the resulting n pairs are pairwise relatively prime. For
example, if n = 3 a solution is 2 + 3 = 5, 4 + 5 = 9, 6 + 1 = 7.
3469 . Proposed by Mihaela Blanariu, Columbia College Chi ago, Chi ago,
IL, USA.
Let p ≥ 2 be a real number. Find the limit
√ √ √
1+ 2(2!)p + 3
3(3!)p + · · · + n n(n!)p
lim .
n→∞ (n!)p
399

3470 . Proposed by Mihaela Blanariu, Columbia College Chi ago, Chi ago,
IL, USA.
Let p ≥ 2 be a real number. Find the limit
√ p √
3
p √ p
1+ 2! + 3! + · · · + n!
n

lim .
n→∞ np+1

3471 . Proposed by Catalin Barbu, Ba au, Romania.


Let ABC be an a ute triangle and M , N , P be the midpoints of the
minor ar s BC , CA, AB ; respe tively. If [XY Z] denotes the area of triangle
XY Z , prove that [M BC]+[N CA]+[P AB] ≥ s(3r−R), where s, r , and R
are the semiperimeter, the inradius, and the ir umradius of triangle ABC ,
respe tively.
3472 . Proposed by Xavier Ros, student, Universitat Polite ni a
 de Catalunya,
Bar elona, Spain.
Let x, y, and z be positive real numbers with x ≤ y ≤ z . Prove that
1+y 1+z 1+x
     
log x · log + log y · log + log z · log ≥ 0.
1+z 1+x 1+y

3473 . Proposed by Walther Janous, Ursulinengymnasium, Innsbru k,


Austria, in memory of Jim Totten.
Let c be a xed real number su h that 0 < c ≤ 1.
(a) For all positive real numbers µ prove that
2 µ 3
√ + p ≤ √ .
cµ + 1 c+µ 2 c+1

(b) ⋆. Determine all positive real numbers λ su h that


λ µ λ+1
√ + p ≤ √
cµ + 1 c + µ2 c+1
holds for all positive real numbers µ.

3474
Gree e.
⋆. Proposed Silouanos Brazitikos and Christos Patilas, Trikala,

Let x, y, q
and z be positive real numbers with xyz = 1. Prove or
disprove that 2 3 xy + yz + z x + x + y + z ≥ 9.


[Ed.: The proposers indi ate that the inequality is supported by om-
puter omputations.℄
400

SOLUTIONS
Au un probleme
 n'est immuable. L'editeur
 est toujours heureux d'en-
visager la publi ation de nouvelles solutions ou de nouvelles perspe tives
portant sur des problemes
 anterieurs.


3363 . [2008 : 362, 364℄ Proposed by Toshio Seimiya, Kawasaki, Japan.


Let ABC be a triangle with ∠ACB = 90◦ + 12 ∠ABC . Let M be the
midpoint of BC . Prove that ∠AM C < 60◦ .
Composite of similar solutions by Ri ardo Barroso Campos, University of
Seville, Seville, Spain; Ri hard I. Hess, Ran ho Palos Verdes, CA, USA; Matti
Lehtinen, National Defen e College, Helsinki, Finland; Madhav R. Modak,
formerly of Sir Parashurambhau College, Pune, India; and Peter Y. Woo,
Biola University, La Mirada, CA, USA.
In △ABC let 4β = ∠B and let W be the foot of the bise tor of ∠A.
We are given that ∠C = 90◦ + 2β , when e ∠A = 90◦ − 6β . We dedu e that
6β < 90◦ , so that β < 15◦ . Moreover, be ause ∠AW C is an external angle
of △ABW , it satis es
∠AW C = 4β + 45◦ − 3β = 45◦ + β < 60◦ .


It remains to show that ∠AM C < ∠AW C . Be ause we are given that ∠C
is obtuse, we have AB > AC , when e the foot W of the angle bise tor
satis es BW > W C . That is, W lies between M and C ; thus ∠AW C is an
external angle of △AM W and therefore satis es ∠AM C < ∠AW C < 60◦ ,
as desired.
Also solved by GEORGE APOSTOLOPOULOS, Messolonghi, Gree e; SEFKET 
ARSLANAGIC,  University of Sarajevo, Sarajevo, Bosnia and Herzegovina; ROY BARBARA,
Lebanese University, Fanar, Lebanon; MICHEL BATAILLE, Rouen, Fran e; CHIP CURTIS,
Missouri Southern State University, Joplin, MO, USA; GERALD EDGECOMB and JULIE
STEELE, students, California State University, Fresno, CA, USA; OLIVER GEUPEL, Bruhl,
 NRW,
Germany; JOHN G. HEUVER, Grande Prairie, AB; WALTHER JANOUS, Ursulinengymnasium,

Innsbru k, Austria; V ACLAV KONECN Y, Big Rapids, MI, USA; KEE-WAI LAU, Hong Kong,
China; TAICHI MAEKAWA, Takatsuki City, Osaka, Japan; THANOS MAGKOS, 3rd High S hool
of Kozani, Kozani, Gree e; SALEM MALIKIC,  student, Sarajevo College, Sarajevo, Bosnia and
Herzegovina; SKIDMORE COLLEGE PROBLEM SOLVING GROUP, Skidmore College, Saratoga
Springs, NY, USA; GEORGE TSAPAKIDIS, Agrinio, Gree e; TITU ZVONARU, Comane  sti,
Romania; and the proposer.

3364 . [2008 : 362, 364℄ Proposed by Toshio Seimiya, Kawasaki, Japan.


Let ABC be a triangle with ∠BAC = 120◦ and AB > AC . Let M be
the midpoint of BC . Prove that ∠M AC > 2 ∠ACB .
401

I.Solution by Peter Y. Woo, Biola University, La Mirada, CA, USA.


Let β = ∠CBA and γ = ∠BCA; then (be ause ∠BAC = 120◦ )
β + γ = 60◦ . Let D be the point of segment BC for whi h ∠BAD = 2β ;
then ∠DAC = 2γ . The Law of Sines tells us that
AD sin 2β
BD = = 2AD cos β ;
sin β

also,
AD sin 2 60◦ − β

= 2AD cos 60◦ − β .

CD = 
sin 60◦ − β
But β+γ = 60◦ and γ > β ; hen e, 60◦ −β > β so that cos 60◦ −β < cos β .


Consequently, CD < BD, when e M lies between B and D. It follows that


∠M AC > ∠DAC = 2∠ACB , as desired.

II. Solution by Matti Lehtinen, National Defen e College, Helsinki, Finland.


Let O be the entre of the ir um ir le Γ of △ABC , and D be the mid-
point of the ar CAB of Γ. Note that A is on the smaller ar DC . Sin e
120◦ = ∠BAC = ∠BDC , DB = DC = DO . Draw the ir le Γ′ with en-
tre D and radius DB (passing through B , O, and C ). Be ause M is the om-
mon midpoint of BC and DO, ir les Γ and Γ′ are symmetri with respe t to
M . Let AM meet Γ′ at E (so that M is also the midpoint of AE ); note that
∠EDC = ∠AOB = 2∠ACB . Extend AE to meet Γ again at F and extend
DE to meet Γ again at G. Be ause the line F A separates D from G and
C , G is on the ar F C opposite D and A; onsequently, ∠F AC > ∠GDC .
But, ∠F AC = ∠M AC , and ∠GDC = ∠EDC = ∠AOB = 2∠ACB , so we
are done.
Also solved by GEORGE APOSTOLOPOULOS, Messolonghi, Gree e; SEFKET 
ARSLANAGIC,  University of Sarajevo, Sarajevo, Bosnia and Herzegovina; ROY BARBARA,
Lebanese University, Fanar, Lebanon; MICHEL BATAILLE, Rouen, Fran e; CHIP CURTIS,
Missouri Southern State University, Joplin, MO, USA; OLIVER GEUPEL, Bruhl,  NRW,

Germany; V ACLAV KONECN  Y,
 Big Rapids, MI, USA; KEE-WAI LAU, Hong Kong, China;
THANOS MAGKOS, 3rd High S hool of Kozani, Kozani, Gree e; SALEM MALIKIC,  student,
Sarajevo College, Sarajevo, Bosnia and Herzegovina; GEORGE TSAPAKIDIS, Agrinio, Gree e;
TITU ZVONARU, Comane  sti, Romania; and the proposer.

3365 . [2008 : 362, 364℄ Proposed by Toshio Seimiya, Kawasaki, Japan.


A square ABCD is ins ribed in a ir le Γ. Let P be a point on the
minor ar AD of Γ, and let E and F be the interse tions of AD with P B
and P C , respe tively. Prove that
,

AE · DF = 2 [P AE] + [P DF ]

where [KLM ] denotes the area of triangle KLM .


402

Solution by John G. Heuver, Grande Prairie, AB.


The altitudes from point P to segments AB and CD lead us to on lude
that
1 P
[P AB] + [P DC] = AB 2 ,
2 A q q .........................................................
..................... .... ....................... ..................
.................. .... ....
D
...................... .. ... b............ .......
............ ......

Furthermore,
. .. . .
............................................................................................................................................
........
E F
.
.
..
... .... .
.
. ...
... ..... ......
.. .. .... .....
.... ..... ... ...
... ...
..
. ... ... ...
... .
.
. ...
.. ... .. .....
1
AB 2 = [P AE] + [AEB] + [P DF ] + [DF C] ,
..
.. .
..... ..
..
... ..
... ... ..
... .... ...
... . .... .
. ...
.... .. .
2
... . ... .
.
. ..
.... ..... ... ... .
.
. ...
... ... ... . ..
.
. ..
... .. ... .. ..

Sin e
...
...
.. ...
.
. ... .
.
.
.
. ...
.
.. . ... . ..
.. ..... ... ... .
.
.
. .
.
.
... .....
b
.. .... ... ..
...
... .... .... ... ... ...
.... .... ... ... ... ....
1
[AEB] + [DF C] = AB(AE + DF ) ,
.... ..... ... ... ....
......... . .. ...
................................................................................................................................
2
..... .....
B C
....... .......
........ . .
...
.
......... ..
.............. .........
............................................

we have

1
AB 2 − [AEB] + [DF C]

[P AE] + [P DF ] =
2
1 1
= AB 2 − AB(AE + DF )
2 2
1 1
=
2
AB(AB − AE − DF ) = AB · EF
2
.
Now, ∠BP C = ∠CP D = ∠45◦ as angles subtended by ar s equal to
a quarter of the ir le. Hen e P F is the bise tor of ∠EP D, and therefore,
EF PE
= .
FD PD
Also, ∠ABP = ∠P DA as angles subtended by the ar AP , so that triangles
P DE and ABE are similar, and therefore,
PE AE
= .
PD AB
Consequently,
EF AE
= ,
FD AB
or EF · AB = AE · DF , and then
1 1
[P AE] + [P DF ] =
2
AB · EF = AE · DF
2
,
whi h ompletes the proof.
Also solved by GEORGE APOSTOLOPOULOS, Messolonghi, Gree e; SEFKET 
ARSLANAGIC,  University of Sarajevo, Sarajevo, Bosnia and Herzegovina; ROY BARBARA,
Lebanese University, Fanar, Lebanon; RICARDO BARROSO CAMPOS, University of Seville,
Seville, Spain; MICHEL BATAILLE, Rouen, Fran e; CHIP CURTIS, Missouri Southern State
University, Joplin, MO, USA; OLIVER GEUPEL, Bruhl, NRW, Germany; WALTHER JANOUS,

Ursulinengymnasium, Innsbru k, Austria; V ACLAV KONECN Y,
 Big Rapids, MI, USA; SALEM
403

MALIKIC,  student, Sarajevo College, Sarajevo, Bosnia and Herzegovina; MADHAV R. MODAK,
formerly of Sir Parashurambhau College, Pune, India; JOEL SCHLOSBERG, Bayside, NY, USA;
D.J. SMEENK, Zaltbommel, the Netherlands; PETER Y. WOO, Biola University, La Mirada,
CA, USA; TITU ZVONARU, Comane  sti, Romania; and the proposer. There was one in orre t
omment submitted.
Kone ny and the proposer ea h began by letting Z be the foot of the perpendi ular from
P to AD, then dedu ing that △DP Z ∼ △BEA and △AP Z ∼ △CF D, whi h then enables
the al ulation of AE · DF .

3366 . [2008 : 362, 365℄ Proposed by Ovidiu Furdui, Campia Turzii, Cluj,
Romania.
Let {x} denote the fra tional part of the real number x; that is,
{x} = x − ⌊x⌋, where ⌊x⌋ is the greatest integer not ex eeding x.
Evaluate Z 1  4
1
dx .
0 x

Solution by Chip Curtis, Missouri Southern State University, Joplin, MO,


USA.
Let I be the integral to be evaluated. Then
N Z 1  4 N Z n+1
X n 1 X {y}4
I = lim dx = lim dy
N →∞
n=1
1
n+1
x N →∞
n=1 n y2

N Z n+1 N
(y − n)4
In ,
X X
= lim dy = lim
N →∞
n=1 n y2 N →∞
n=1

where
n+1
(y − n)4
Z
In = dy .
n y2
We have
n+1
4n3 n4
Z  
2 2
In = y − 4ny + 6n − + 2 dy
n y y
 3 n+1
4
y n
= − 2ny 2 + 6n2 y − 4n3 ln y −


3 y
n
4
1 n 1
 
2 3 3
= 3n − n + − + n − 4n ln 1 +
3 n+1 n
1 1 1
   
2 2 3
= 3n − n + + n − n − + 1 − 4n ln 1 +
3 n+1 n
4 1 1
 
= 4n2 − 2n + − − 4n3 ln 1 + .
3 n+1 n
404

Substituting the series expansions


∞ ∞
x (−1)k+1 xk
and ,
X X
= (−1)k xk+1 ln(1 + x) =
x+1 k=0 k=1
k
1
with x = n
(n ≥ 2) into the last expression for In yields
∞ ∞
4 X (−1)k X (−1)k+1
In = 4n2 − 2n + − − 4n3
3 k=0
nk+1 k=1
knk
4 1 1 1
 
= 4n2 − 2n + − 4n3 − 2
+
3 n 2n 3n3
∞ ∞
X (−1)k X (−1)k+1
− − 4n3
k=0
nk+1 k=4
knk
∞ ∞
X (−1)k X (−1)k+1
= − − 4n3
k=0
nk+1 k=4
knk
∞ ∞
X (−1)k X (−1)k+1
= − 4
k=1
nk k=4
knk−3
∞ ∞
X (−1)k X (−1)k
= − 4
k=1
nk k=1
(k + 3)nk
∞  k ∞
X 1 X (−1)k
= − − 4
k=2
n k=2
(k + 3)nk

1 (−1)k
,
X
= − 4
n(n + 1) k=2
(k + 3)nk
and the last formula is valid for I1 as well. Thus,
∞ ∞ X ∞
1 1 1 k
X X  
I = − 4 −
n=1
n(n + 1) n=1 k=2
k+3 n
∞ ∞
X (−1)k X 1
= 1 − 4
k=2
k+3 n=1
nk

(−1)k
ζ(k) .
X
= 1 − 4
k=2
k+3
Also solved by WALTHER JANOUS, Ursulinengymnasium, Innsbru k, Austria; RICHARD
I. HESS, Ran ho Palos Verdes, CA, USA; OLIVER GEUPEL, Bruhl, NRW, Germany; MADHAV
R. MODAK, formerly of Sir Parashurambhau College, Pune, India; PAOLO PERFETTI,
Dipartimento di Matemati a, Universita degli studi di Tor Vergata Roma, Rome, Italy; and
the proposer. There were two in orre t solutions submitted.
The proposer obtained the answer − 31 − γ + 2 ln 2π − 12 ln A + 12 ln B, where γ is
Euler's onstant and where
2 2 2
11 22 · · · nn 11 22 · · · nn
A = lim
n→∞ n2 n 1 −n2
; B = lim
n→∞ n3 n2 n −n3 n
n 2 + 2 + 12 e 4 n 3 + 2 +6 e 9 + 12
405

are the Glaisher{Kinkelin onstants of order 1 and order 2, respe tively.


ζ(k) − 1

Geupel obtained I = 0.14553289 . . . , whi h he
P
= 24
k(k + 1)(k + 2)(k + 3)
k=2
∞ (−1)k ζ(k)
remarked onverges more rapidly than the series 1 − and is thus better suited
P
k=2 k+3
for numeri al approximation.
Janous reports the generalization
 
1  N N −2  
1 N  1 1
Z  
N −j −1
X
j
dx =  (−1) 2 − 1  − (−1)N N ln 2 −
0 x j =0
j N −j −1 2

(−1)p (p − 1)
ζ(p) − 1 .
X 
+
p=2
p + N − 1

Keith Ekblaw, Walla Walla, WA, USA, obtained the estimate I ≈ 0.146 by a Monte Carlo
approa h, whi h is orre t to 3 de imal pla es after rounding.

3367 . [2008 : 362, 365℄ Proposed by Li Zhou, Polk Community College,


Winter Haven, FL, USA.
Let p(x) = an xn + an−1 xn−1 + · · · + a1 x be a polynomial with integer
n
oeÆ ients, where an > 0 and ak = 1. Prove or disprove that there are
P
k=1
in nitely many pairs of positive integers (k, ℓ) su h that p(k + 1) − p(k) and
p(ℓ + 1) − p(ℓ) are relatively prime.

Solution by Cristinel Morti i, Valahia University of Targoviste, Romania,


modi ed slightly by the editor.
Let Q(x) = p(x + 1) − p(x). Then we have
n
ak − 0 = 1 .
X
Q(0) = p(1) − p(0) =
k=1

Hen e, Q(x) = xq(x) + 1 for some polynomial q(x) of degree n − 1.


We need to nd in nitely many positive integers k and l su h that
gcd Q(k), Q(l) = 1.
Sin e the leading term in Q(x) is nan xn−1 we have Q(k) > 0 for
suÆ iently large positive integers k. For su h k let l = Q(k) = kq(k) + 1,
so that l is a positive integer. If, on the ontrary, gcd Q(k), Q(l) 6= 1, then
there is a prime number p su h that p | Q(k) and p | Q(l). Then, sin e
Q(l) = lq(l) + 1 = Q(k)q(l) + 1, we on lude that p | 1, a ontradi tion.
Also solved by ROY BARBARA, Lebanese University, Fanar, Lebanon; OLIVER GEUPEL,
Bruhl,
 NRW, Germany; WALTHER JANOUS, Ursulinengymnasium, Innsbru k, Austria; and the
proposer.
Morti i remarked that the on lusion in fa t holds for any polynomial Q(k) whose on-
stant term is either 1 or −1. This is lear from the proof featured above. Barbara proved the
n
stronger results that (1) if an 6= 0 and ak = 1, then there are in nitely many positive
P
k=1
406

integers k su h that p(k + 1) − p(k) are all pairwise relatively prime, and (2) if f(x) is a
polynomial with integer oeÆ ients and positive degree su h that f(0) = 1, then there are
in nitely many integers k1 < k2 < k3 < · · · su h that the f(ki )'s are all pairwise relatively
prime.

3368 . [2008 : 363, 365℄ Proposed by Neven Juri , Zagreb, Croatia.


Let m be an integer, m ≥ 2, and let A = [Aij ] be a blo k matrix of
dimension 2m × 2m with Aij ∈ M4,4 (N) for 1 ≤ i, j ≤ 2m−2 , de ned by
Aij = 2m Bij + Cij , where

2m − 4i + 4 2m − 4i + 4
 
4i − 4 4i − 4
 4i − 3 m m
2 − 4i + 3 2 − 4i + 3 4i − 3 
Bij = 
 4i − 2 2m − 4i + 2 2m − 4i + 2 4i − 2 
 ,
2m − 4i + 1 4i − 1 4i − 1 2m − 4i + 1
 
4 − 4j 4j − 2 4j − 1 1 − 4j
4j − 3 3 − 4j 2 − 4j 4j 
and Cij = 
4j − 3 3 − 4j 2 − 4j 4j 
.

4 − 4j 4j − 2 4j − 1 1 − 4j

Show that matrix A is a magi square of order 2m .


Solution by Oliver Geupel, Bruhl,
 NRW, Germany.
For onvenien e, write Bij = B (i) = b(i) and Cij = C (j) = ckℓ ,
   
(j)
kℓ
and note that these 4 × 4 matri es have integer entries.
Claim 1 For ea h entry a of A, 1 ≤ a ≤ 22m .
Proof: We have a = 2m b(i) kℓ + ckℓ for some i, j , k, and ℓ. If we also have
(j)

(k, ℓ) ∈ {(1, 2), (1, 3)}, then bkℓ ≥ 0 and ckℓ ≥ 1; hen e a ≥ 1. Otherwise,
(i) (j)

bkℓ ≥ 1 and ckℓ ≥ 1−2m ; hen e a ≥ 2m +1−2m = 1. This proves the lower
(i) (j)

bound. For the upper bound, we observe that for (k, ℓ) ∈ {(1, 1), (1, 4)} we
have b(i)
kℓ ≤ 2
m
and c(j)
kℓ ≤ 0; hen e a ≤ 2
m
· 2m = 22m . In any other ase
bkℓ ≤ 2 − 1 and ckℓ ≤ 2 ; so again a ≤ 2 2m − 1 + 2m = 22m .
(i) m (j) m m


Claim 2 The entries of A are distin t.


Proof: If two entries of A are equal then 2m b(i) kℓ + ckℓ = 2 bkℓ + ckℓ for
′ ′
(j) m (i ) (j )

some j , j ′ , k, k′ , ℓ, and ℓ′ . Let b = b(i)


kℓ , b = bk ℓ , c = ckℓ , and c = ck ℓ .
′ ′
′ (i ) (j) ′ (j )
′ ′ ′ ′

We then have 1 ≤ |c − c′ | ≤ 4 · 2m−2 − 1 − 4 · 2m−2 = 2m+1 − 1 < 2m+1 .


Without loss of generality, suppose that c < c′ . Then c′ − c = 2m and
b = b′ + 1, hen e c ≡ c′ (mod 4) and b ≡ b′ + 1 (mod 4). There are four
ases for c and c′ :
Case 1 (k, ℓ) ∈ {(1, 1), (4, 1)} and (k , ℓ ) ∈ {(2, 4), (3, 4)};
′ ′

Case 2 (k, ℓ) ∈ {(1, 4), (4, 4)} and (k , ℓ ) ∈ {(2, 1), (3, 1)};
′ ′
407

Case 3 (k, ℓ) ∈ {(2, 3), (3, 3)}


and (k′ , ℓ′ ) ∈ {(1, 2), (4, 2)};
Case 4 (k, ℓ) ∈ {(2, 2), (3, 2)}
and (k′ , ℓ′ ) ∈ {(1, 3), (4, 3)};
ea h of them in ompatible with the ondition b ≡ b′ + 1 (mod 4).

Claim 3 The sum of the entries along any horizontal, verti al, or main diagonal
line of A is 23m−1 + 2m−1 .
Proof: The sum along any given horizontal or verti al line of B (i) and C (j) is
SB = 2m+1 and SC = 2, respe tively. Therefore, the sum of the entries in
ea h line of A is 2m−2 2m SB +SC = 2m−2 2m ·2m+1 +2 = 23m−1 +2m−1 .


The main diagonal sums of B (i) and C (j) are b(i) = 2m+2 − 16i + 10 and
c(j) = 10 − 16j . The entries in ea h main diagonal of A then also have the
2m−2 2m−2
sum 2m b(i) + c(j) = 23m−1 + 2m−1 .
P P
i=1 j=1

Also solved by the proposer. One in omplete solution was re eived that veri ed the row,
olumn, and diagonal sums, but did not show that the entries of the magi square onsisted of
1, 2, . . . , 22m .

3369 . [2008 : 363, 365℄ Proposed by George Tsintsifas, Thessaloniki,


Gree e.
Let A1 A2 A3 A4 be a tetrahedron whi h ontains the entre O of its
ir umsphere as an interior point. Let ρi be the distan e from O to the fa e
opposite vertex Ai . If R is the radius of the ir umsphere, prove that
4
4
ρi .
X
R ≥
3 i=1

Solution by Oliver Geupel, Bruhl,


 NRW, Germany.
The laim is false; the following ounterexample is adapted from [1℄.
First, onsider the degenerate tetrahedron A′1 A′2 A′3 A′4 , where
A′1 = A′2 = (−1, 1, 0) ,
A′3 = (1, 1, 0) ,
A′4 = (−1, −1, 0) ,

O = (0, 0, 0), and R = 2. We have ρ1 = ρ2 = 0, and ρ3 = ρ4 = 1, so
that √
4R 4 2 √
4
= = 2 2 < 3.
P 2
ρi
i=1
408

Now, let
−1 + ε, 1 − ε, 2ε(2 − ε) ,
 p 
A1 =

−1 + ε, 1 − ε, − 2ε(2 − ε) ,
 p 
A2 =
√  √
π π
−ε , 0 ,
   
A3 = 2 cos − ε , 2 sin
√ 4  √ 4
3π 3π
.
  
A4 = 2 cos − + ε , 2 sin − +ε , 0
4 4
It is easy to verify that point O is in the interior of the tetrahedron A1 A2 A3 A4 ,
and that Ai → A′i for ea h i as ε → 0, as well as
4R √
4
−→ 2 2 .
P
ρi
i=1

The bound 2 2 is the best possible. This is a orollary from the fol-
lowing:
Theorem [1℄. Let A1 A2 A3 A4 be a nondegenerate tetrahedron whose ir um-
entre O is not an exterior point. Let P be a point not exterior to A1 A2 A3 A4 .
Let the distan es from P to the verti es and to the fa es of A1 A2 A3 A4 be
denoted by Ri and ρi , respe tively. Then
4
P
Ri

i=1
4
> 2 2,
P
ρi
i=1

and 2 2 is the greatest lower bound.
Referen es

[1℄ Ni holas D. Kazarino , \D.K. Kazarino 's inequality for tetrahedra",


Mi higan Mathemati al Journal, Vol. 4, No. 2 (1957), pp. 99-104.
Counterexample also given by Peter Y. Woo, Biola University, La Mirada, CA, USA.
George Apostolopoulos, Messolonghi, Gree e, ited the book by Kazarino , Geometri
Inequalities, Yale University Press, 1961, p. 116.
No omplete solutions or other omments were submitted.

3370 . [2008 : 363, 365℄ Proposed by George Tsintsifas, Thessaloniki,


Gree e.
Let ai and bi be positive real numbers for 1 ≤ i ≤ k, and let n be a
positive integer. Prove that
k
!n k
! k
!n−1
1 ai 1
.
X X X
ai
n
≤ bi
n−1

i=1 i=1
bi i=1
409

Composite of similar or identi al solutions submitted by all the solvers whose


names appear below (ex ept the two solvers identi ed by a \*" before their
names).
n 1 1
Let p = n and q = n−1
. Then p > 1, q > 1 and p
+ = 1.
q
[Ed: learly, n > 1 for the given inequality to make sense.℄
1

Let xi = ab i and yi = bi . Then xi and yi are positive for ea h i.


  n 1
n

i
k
 k
 p1  k
 q1
By Holder's
 Inequality, we have xpi yiq whi h
P P P
xi yi ≤
i=1 i=1 i=1
1 n−1
k k k
  
n
 n
1
ai
be omes bn−1 .
P P P
ain ≤ i
i=1 i=1 bi i=1
The result follows by raising both sides to the nth power.
Also solved by GEORGE APOSTOLOPOULOS, Messolonghi, Gree e; SEFKET 
ARSLANAGIC,  University of Sarajevo, Sarajevo, Bosnia and Herzegovina; MICHEL BATAILLE,
Rouen, Fran e; MANUEL BENITO, OSCAR CIAURRI, EMILIO FERNANDEZ, and LUZ RONCAL,
Logrono,
~ Spain; CAO MINH QUANG, Nguyen Binh Khiem High S hool, Vinh Long, Vietnam;
CHIP CURTIS, Missouri Southern State University, Joplin, MO, USA; OLIVER GEUPEL, Bruhl, 
NRW, Germany; JOE HOWARD, Portales, NM, USA; WALTHER JANOUS, Ursulinengymnas-
ium, Innsbru k, Austria; SALEM MALIKIC,  student, Sarajevo College, Sarajevo, Bosnia and
Herzegovina; DUNG NGUYEN MANH, High S hool of HUS, Hanoi, Vietnam; *CRISTINEL
MORTICI, Valahia University of Targoviste, Romania; PAOLO PERFETTI, Dipartimento di
Matemati a, Universita degli studi di Tor Vergata Roma, Rome, Italy; JOEL SCHLOSBERG,
Bayside, NY, USA; *PETER Y. WOO, Biola University, La Mirada, CA, USA; TITU ZVONARU,
Comane
 sti, Romania; and the proposer.

3371 . [2008 : 363, 368℄ Proposed by George Tsintsifas, Thessaloniki,


Gree e.
Let ABC be a triangle with a, b, and c the lengths of the sides opposite
the verti es A, B , and C , respe tively, and let M be an interior point of
△ABC . The lines AM , BM , and CM interse t the opposite sides at the
points A1 , B1 , and C1 , respe tively. Lines through M perpendi ular to the
sides of △ABC interse t BC , CA, and AB at A2 , B2 , and C2 , respe tively.
Let p1 , p2 , and p3 be the distan es from M to the sides BC , CA, and AB ,
respe tively. Prove that
[A1 B1 C1 ] (ap1 + bp2 )(bp2 + cp3 )(cp3 + ap1 ) a b c
 
= + + ,
[A2 B2 C2 ] 8a2 b2 c2 p1 p2 p3

where [KLM ] denotes the area of triangle KLM .


Solution by Joel S hlosberg, Bayside, NY, USA.
By the basi formula for the area of a triangle, [ACM ] = 12 bp2 and
[CM B] = 12 ap1 . It is well known that if segments Y Z and Y ′ Z ′ lie on the
410

same line and X is any point, then [XY Z] : [XY ′ Z ′ ] = Y Z : Y ′ Z ′ , so that


[ACC1 ] [CC1 B] CC1
= = ;
[ACM ] [CM B] CM
[ACC1 ] [ACM ] AC1 bp2
= = = .
[CC1 B] [CM B] C1 B ap1

By similar reasoning, BA 1
A1 C
=
cp3
bp2
and CBB1 A
1
=
ap1
cp3
. A known formula (see
Eri W. Weisstein, \Routh's Theorem," at http://mathworld.wolfram.com/
RouthsTheorem.html) states that if A′ , B ′ , and C ′ are points on the sides
BC , CA, and AB of △ABC , respe tively, then
AC ′ BA′ CB ′
· · + 1
[A′ B ′ C ′ ] =  ′
C ′ B A′ C B ′ A

  [ABC] .
AC BA CB ′
+1 +1 +1
C′B A′ C B′A

Therefore,
bp2 cp3 ap1
· · +1
ap1 bp2 cp3
[A1 B1 C1 ] =     [ABC]
bp2 cp3 ap1
+1 +1 +1
ap1 bp2 cp3

2abcp1 p2 p3
= [ABC] .
(ap1 + bp2 )(bp2 + cp3 )(cp3 + ap1 )
Sin e ∠M A2 C and ∠M B2 C are right angles,
∠A2 M B2 = 360◦ − ∠M A2 C − ∠M B2 C − ∠A2 CB2
= 180◦ − ∠ACB ,
so sin ∠A2 M B2 = sin C , and sin e all four angles of quadrilateral A2 M B2 C
are less than 180◦ , A2 M B2 C is onvex. The well-known area formula for a
triangle, [XY Z] = 21 XY · XZ sin ∠Y XZ , yields
1
[M A2 B2 ] = M A2 · M B2 sin ∠A2 M B2
2
p1 p2 1 p1 p2
 
= ab sin C = [ABC] .
ab 2 ab

By similar reasoning, [M B2 C2 ] = pbc


2 p3 p p
[ABC] and [M C2 A2 ] = 3 1 [ABC],
ca
and B2 M C2 A and C2 M A2 B are onvex. Sin e A2 M B2 C , B2 M C2 A, and
C2 M A2 B are onvex, M is outside of △A2 B2 C , △B2 C2 A, and △C2 A2 B ,
and sin e M is in the interior of △ABC , M must be in the interior of
△A2 B2 C2 . Therefore,
[A2 B2 C2 ] = [M A2 B2 ] + [M B2 C2 ] + [M C2 A2 ] ,
411

and so
p1 p2 p2 p3 p3 p1
 
[A2 B2 C2 ] = + + [ABC]
ab bc ca
p1 p2 p3 a b c
 
= + + [ABC] .
abc p1 p2 p3
Finally,
 
p1 p2 p3
a b c
+ + [ABC]
[A2 B2 C2 ] abc p1 p2 p3
=
[A1 B1 C1 ] 2abcp1 p2 p3
[ABC]
(ap1 + bp2 )(bp2 + cp3 )(cp3 + ap1 )

(ap1 + bp2 )(bp2 + cp3 )(cp3 + ap1 ) a b c


 
= + + .
2a2 b2 c2 p1 p2 p3
Also solved by GEORGE APOSTOLOPOULOS, Messolonghi, Gree e; ARKADY ALT, San
Jose, CA, USA; MICHEL BATAILLE, Rouen, Fran e; CHIP CURTIS, Missouri Southern State
University, Joplin, MO, USA; OLIVER GEUPEL, Bruhl,  NRW, Germany; WALTHER JANOUS,
Ursulinengymnasium, Innsbru k, Austria; PETER Y. WOO, Biola University, La Mirada, CA,
USA; and TITU ZVONARU, Comane  sti, Romania. There was one in orre t solution submitted.
Both Bataille and Geupel al ulated with bary entri oordinates. Geupel also used the
formula for the area of the pedal triangle of M , to wit [A2 B2 C2 ] = |R2 −OM 2 |/4R2 , where
O and R are the ir um entre and ir umradius of triangle ABC, respe tively.

3372. [2008 : 364, 366℄ Proposed by Vo Quo Ba Can, Can Tho University
of Medi ine and Pharma y, Can Tho, Vietnam.
If x, y, z ≥ 0 and xy + yz + zx = 1, prove that

1 1 1 2 6
(a) p + p + p ≥ ;
2x2 + 3yz 2y 2 + 3zx 2z 2 + 3xy 3
1 1 1 √
(b)⋆ p
2
+ p
2
+ p
2
≥ 2 2 .
x + yz y + zx z + xy

Solution by George Apostolopoulos, Messolonghi, Gree e.


(a) Sin e 2x2 ≤ 3x2 we have 2x2 + 3yz ≤ 3x2 + 3yz , thus
1 1 1
p
2
≥ p
2
= √ p .
2x + 3yz 3x + 3yz 3 x2 + yz
Similar inequalities hold for the other two terms on the left side of the desired
inequality, and we now have
1 1 1
p + p + p
2
2x + 3yz 2
2y + 3zx 2
2z + 3xy
!
1 1 1 1
≥ √ p + p + p .
3 x2 + yz y 2 + zx z 2 + xy
412

The desired inequality follows now from (b), whi h is proven below.
(b)⋆ More generally, we will prove that if x, y, z ≥ 0 and xy+yz+zx > 0,
then

1 1 2 2 1
p + p + p ≥ √ . (1)
x2 + yz y 2 + zx z 2 + xy xy + yz + zx

Sin e this is symmetri in x, y, z , we may assume that x ≥ y ≥ z. Noti e


that √
1 1 2 2
p
2
+ p
2
≥ p .
y + zx z + xy y + z 2 + xy + zx
2
r
[Ed: we have p 1 + p
1
≥ 2 p
1
p
1
from the
y 2 + zx z 2 + xy y 2 + zx
z 2 + xy
1 2
AM{GM Inequality, and also p p ≥ 2
y + zx + z 2 + xy
; the
y 2 + zx z 2 + xy
inequality above now follows from these two.℄
So it suÆ es to prove that
√ √
1 2 2 2 2
p + p ≥ √ .
x2 + yz y 2 + z 2 + xy + zx xy + yz + zx

Let K = xy + yz + zx and L = y 2 + z 2 + xy + zx. Then


√ √ √ √ √  √
2 2 y 2 − yz + z 2

2 2 2 2 2 2 L− K
√ − √ = √ = √ √ √  .
K L KL KL L + K

It is lear that L ≥ K , L ≥ 2 y2 − yz + z 2 , and K ≥ y x2 + yz .


 p

[Ed: Sin e x ≥ y ≥ z , we have L = y2 + z 2 + xy + zx ≥ y2 + z 2


+ y 2 + z 2 ≥ 2(y 2 − yz +pz 2 ) and K = xy + yz p + zx ≥ xy + yz + yz

= y x2 + 4xz + 4z 2 ≥ y x2 + 4yz + 4z 2 ≥ y x2 + yz .℄
Thus,
√ √ √
2 2 y 2 − yz + z 2 2 2 y 2 − yz + z 2 2 y 2 − yz + z 2
  
√ √ √ ≤ √ √ = √
KL( L + K) KL(2 K) K L

2 y 2 − yz + z 2

≤ p q 
y x2 + yz 2 y 2 − yz + z 2
p p
y 2 − yz + z 2 y2
= p ≤ p
y x2 + yz y x2 + yz
1
= p ,
x2 + yz
413

and we have proved (1).


Sin e xy + yz + zx = 1, inequality (b) now follows from (1).
Part (a) also solved by Oliver Geupel, Bruhl,
 NRW, Germany; and the proposer.
Geupel notes that the problem is Problem 1.89 on page 74.f with solution in the e-paper
toanhocmuonmaumain.pdf written by the proposer in Vietnamese, available on the MathLinks
forum website http://www.mathlinks.ro/viewtopic.php?t=197674.

3373. Proposed by Vo Quo Ba Can, Can Tho University of Medi ine and
Pharma y, Can Tho, Vietnam.
Let x, y, z , and t be positive real numbers. Prove that
(x + y)(x + z)(x + t)(y + z)(y + t)(z + t) ≥ 4xyzt(x + y + z + t)2 .

Solution by Cristinel Morti i, Valahia University of Targoviste, Romania.


Dividing by xyzt(x + z)(y + t)(x + t)(y + z) the inequality be omes

x+y z+t 2(x + y + z + t) 2(x + y + z + t)


· ≥ · .
xy zt (x + z)(y + t) (x + t)(y + z)

Thus we need to show:


2 2 2 2 1 1 1 1
     
+ + ≤ + + .
x+z y+t x+t y+z x y z t

By the AM{GM Inequality we have

2 2 2 2 1 1 1 1
     
+ + ≤ √ +√ √ +√ ,
x+z y+t x+t y+z xz yt xt yz

while by the Cau hy-S hwartz Inequality we have


s
1 1 1 1 1 1
 
√ +√ ≤ + +
xz yt x y z t
and s
1 1 1 1 1 1
 
√ +√ ≤ + + .
xt yz x y z t
Combining the last three inequalities we obtain the desired inequality.
Also solved by GEORGE APOSTOLOPOULOS, Messolonghi, Gree e; SEFKET 
ARSLANAGIC,  University of Sarajevo, Sarajevo, Bosnia and Herzegovina; MICHEL BATAILLE,
Rouen, Fran e; CHIP CURTIS, Missouri Southern State University, Joplin, MO, USA; OLIVER
GEUPEL, Bruhl,
 NRW, Germany; JOE HOWARD, Portales, NM, USA; WALTHER JANOUS,
Ursulinengymnasium, Innsbru k, Austria; SALEM MALIKIC,  student, Sarajevo College,
414

Sarajevo, Bosnia and Herzegovina; KHANH BAO NGUYEN, High S hool for Gifted Stu-
dents, Hanoi University of Edu ation, Hanoi, Vietnam; PAOLO PERFETTI, Dipartimento di
Matemati a, Universita degli studi di Tor Vergata Roma, Rome, Italy; PETER Y. WOO, Biola
University, La Mirada, CA, USA; TITU ZVONARU, Comane  sti, Romania; and the proposer.
Howard notes that this problem is similar to CRUX 2393(a) [1999 : 447℄. In fa t, Maliki
observes that by using 2393(a) for (a, b, c, d) = (x, y, z, t), (a, b, c, d) = (x, z, t, y), and
(a, b, c, d) = (x, t, y, z) (that is, all the y li permutations of y, z, t) and multiplying the
resulting inequalities together we get exa tly the desired inequality.
Geupel notes that the problem is Problem 1.73 on page 64.f with solution in the e-paper
toanhocmuonmaumain.pdf written by the proposer in Vietnamese, available on the MathLinks
forum website http://www.mathlinks.ro/viewtopic.php?t=197674.

3374 . [2008 : 364, 366℄ Proposed by Pham Huu Du , Ballajura, Australia.


Let a, b, and c be positive real numbers. Prove that
a2 b2 c2 a+b+c
+ + ≤ √ .
a2 + bc b2 + ca c2 + ab 3
2 abc

Solution by Peter Y. Woo, Biola University, La Mirada, CA, USA.


By homogeneity, we may take abc = 1 and a ≤ b ≤ c. Also, if a = 1,
then a = b = c = 1 and there is nothing to prove, so we take a < 1. x
For onvenien e, write x = a3 , y = b3 , and z = c3 ; let f (x) = x + 1 ,
and let the left and right sides of the inequality be L = f (x) + f (y) + f (z)
and R = 12 x1/3 + y1/3 + z 1/3 , respe tively.



r
z
Setting m = yz and r =
y
, we have that m >1 (be ause a <1
m
implies that yz > 1), y = r
, and z = mr. We then nd that
2m m mr
2f (m) − f (y) − f (z) = − −
m+1 m+r mr + 1
m(m − 1)(r − 1)2
= ≥ 0,
(m + 1)(m + r)(mr + 1)
hen e L annot de rease if ea h of y and z are repla ed by their geometri

mean. On the other hand, from bc ≤ b + 2
c
, we see that R annot in rease
if y and z are ea h repla ed by their geometri mean.
Therefore, it suÆ es to prove the inequality under the additional as-
sumption that b = c and a = b12 . This new relation yields
(1 + 2y) y 3 + y 2 + y + 1 − 4b11 − 6b5 − 2b2

R−L =  ,
2b2 y 3 + y 2 + y + 1
where the denominator is positive and (after some omputation) the numer-
ator be omes
(b − 1)2 (b − 1) 2b9 + 2b8 − b6 − b5 + 5b4 + 3b3 + b2 + 2b + 1 ,
  
415

whi h is also a positive quantity sin e b > 1.


Thus, R − L > 0 in this last ase, and the proof is omplete.
Also solved by GEORGE APOSTOLOPOULOS, Messolonghi, Gree e; CHIP CURTIS,
Missouri Southern State University, Joplin, MO, USA (2 solutions); WALTHER JANOUS,
Ursulinengymnasium, Innsbru k, Austria; KEE-WAI LAU, Hong Kong, China; and the proposer.
There were two in orre t solutions submitted.

3375 . Proposed by Ovidiu Furdui, Campia Turzii, Cluj, Romania.


Let p be a non-negative integer and x any real number. Find the sum

!
x x2 xn+p
.
X
n x
(−1) e −1− − − ··· −
n=1
1! 2! (n + p)!

Solution by Cristinel Morti i, Valahia University of Targoviste, Romania.


If r > 0 and f is a fun tion with

an xn , x ∈ (−r, r) ,
X
f (x) =
n=0

then we will show that



X
(−1)n f (x) − a0 − a1 x − · · · − an+p xn+p

n=1

p/2
f (x) + f (−x)
, even ,
 X
a2i x2i − p



2

(1)

i=0
= (p−1)/2
 f (x) − f (−x)
, odd .
X
a2i+1 x2i+1 −

p




i=0
2

Taking f (x) = ex , we obtain the answer



!
X
n x xn+p
(−1) f (x) − 1 − − ··· −
n=1
1! (n + p)!

p/2
x2i ex + e−x
, even,
 X
− p



(2i)! 2


i=0
= (p−1)/2
 x2i+1 ex − e−x
, odd.
 X
− p




i=0
(2i + 1)! 2

To prove (1), note that the general term


an = (−1)n f (x) − a0 − a1 x − · · · − an+p xn+p

416

onverges to zero, so it suÆ es to nd the limit of the sequen e {s2n }, where


n
ak ,
X
sn =
k=1

be ause s2n+1 = s2n + a2n+1 and a2n+1 → 0. We have


n n
.
X X
−a2k+p x2k+p

s2n = (a2k−1 + a2k ) =
k=1 k=1

If p is even, then
p/2 n+p/2
a2i x2i ;
X X
s2n = a2i x2i −
i=0 i=0

while if p is odd, then


(p−1)/2 n+(p−1)/2
a2i+1 x2i+1 .
X X
s2n = a2i+1 x2i+1 −
i=0 i=0

Now the relation (1) follows from the equations


∞ ∞
f (x) + f (−x) f (x) − f (−x)
; .
X X
a2i x2i = a2i+1 x2i+1 =
i=0
2 i=0
2

Also solved by MICHEL BATAILLE, Rouen, Fran e; MANUEL BENITO, OSCAR CIAURRI,
EMILIO FERNANDEZ, and LUZ RONCAL, Logrono, ~ Spain; CHIP CURTIS, Missouri Southern
State University, Joplin, MO, USA; OLIVER GEUPEL, Bruhl,
 NRW, Germany; RICHARD I. HESS,
Ran ho Palos Verdes, CA, USA; WALTHER JANOUS, Ursulinengymnasium, Innsbru k, Austria;
and the proposer. There were three in orre t solutions submitted.

Crux Mathemati orum


with Mathemati al Mayhem
Former Editors / An iens Reda teurs:
 Bru e L.R. Shawyer, James E. Totten

Crux Mathemati orum


Founding Editors / Reda teurs-fondateurs:
 Leopold
 Sauve & Frederi k G.B. Maskell
Former Editors / An iens Reda teurs:
 G.W. Sands, R.E. Woodrow, Bru e L.R. Shawyer

Mathemati al Mayhem
Founding Editors / Reda teurs-fondateurs:
 Patri k Surry & Ravi Vakil
Former Editors / An iens Reda teurs:
 Philip Jong, Je Higham, J.P. Grossman,
Andre Chang, Naoki Sato, Cyrus Hsia, Shawn Godin, Je Hooper

You might also like